Косинус на окружности: Тригонометрические функции на единичной окружности. Синус и косинус — урок. Алгебра, 10 класс.

Содержание

основное тригонометрическое тождество, знаки синусов и косинусов

  1. Синус и косинус острого угла в прямоугольном треугольнике
  2. Основное тригонометрическое тождество
  3. Синус и косинус угла на числовой окружности
  4. Знаки синусов и косинусов
  5. Синусы и косинусы углов πk/2
  6. Синусы и косинусы углов π/4+πk/2
  7. Синусы и косинусы углов π/6+πk/2
  8. Синусы и косинусы углов π/3+πk/2
  9. Примеры

п.1. Синус и косинус острого угла в прямоугольном треугольнике

Синус острого угла в прямоугольном треугольнике равен отношению противолежащего катета к гипотенузе.
sinα=$\frac{a}{c} $
Косинус острого угла в прямоугольном треугольнике равен отношению прилежащего катета к гипотенузе.
cosα=$\frac{b}{c} $

Например:

B ΔABC, ∠C = 90°, a = 2, b = 4. Найдем синус и косинус ∠A. 2\alpha=1 $$

п.3. Синус и косинус угла на числовой окружности

Числовая окружность расположена в декартовой прямоугольной системе координат.
Отметим на числовой окружности точку M, где луч OM составляет с положительным направлением оси OX угол α. Найдем координаты точки M.
Рассмотрим ΔMOK.
MKO=90°, ∠MOK
OM=1 – гипотенуза
По определению синуса и косинуса острого угла в прямоугольном треугольнике получаем: \begin{gather*} cos\alpha=\frac{OK}{OM}=\frac{x}{1}=x\\ sin\alpha=\frac{MK}{OM}=\frac{y}{1}=y \end{gather*}

Каждому углу α на числовой окружности соответствует точка, координаты которой: $$ x=cos\alpha,\ \ y=sin\alpha $$

Уравнение числовой окружности по определению: x2 + y2 = 1
Откуда снова получаем основное тригонометрическое тождество:

sin2α + cos2α = 1

п.

4. Знаки синусов и косинусов
Знак косинуса — это знак координаты x для точки на числовой окружностиЗнак синуса — это знак координаты
y
для точки на числовой окружности
\begin{gather*} cos\alpha\gt 0,\ \ \text{если}\ -\frac{\pi}{2}\lt\alpha \lt \frac{\pi}{2}\\ cos\alpha\lt 0,\ \ \text{если}\ \frac{\pi}{2}\lt\alpha\lt \frac{3\pi}{2} \end{gather*}\begin{gather*} sin\alpha\gt 0,\ \ \text{если}\ 0\lt \alpha \lt \pi\\ sin\alpha\lt 0,\ \ \text{если}\ \pi\lt \alpha \lt 2\pi \end{gather*}

п.5. Синусы и косинусы углов \(\frac{\pi k}{2}\)

Базовыми точками на числовой окружности для углов, кратных прямому углу (углы \(\frac{\pi k}{2}\)), будут четыре точки: 0°, 90°, 180°, 270° \(\left(0,\ \frac{\pi}{2},\ \pi,\ \frac{3\pi}{2}\right)\).
Все остальные точки (например, 360°, 900° или –540°) будут отличаться от базовых точек на один или несколько полных периодов 2πk, т. е. будут совпадать с ними на окружности.
Синусы и косинусы для совпадающих точек равны. Косинус – это координата x, синус – координата y.

α90°180°270°
0π/2π3π/2
xcosα10–10
ysinα010–1

п.6. Синусы и косинусы углов \(\frac{\pi}{4}+\frac{\pi k}{2}\)

Базовыми точками на числовой окружности для угла 45° и всех отстоящих от него на углы, кратные прямому (углы \(\frac{\pi}{4}+\frac{\pi k}{2}\)), будут четыре точки: 45°, 135°, 225°, 315° \(\left(\frac{\pi}{4},\ \frac{3\pi}{4},\ \frac{5\pi}{4},\ \frac{7\pi}{4}\right)\).
Все остальные точки (например, 405°, 945° или –585°) будут отличаться от базовых точек на один или несколько полных периодов 2
πk
, т. 2=\frac12 \overset{m\gt 0}{\Rightarrow} m=\frac{1}{\sqrt{2}}=\frac{\sqrt{2}}{2} $$ Все исследуемые точки на числовой окружности будут иметь пару координат \(\frac{\sqrt{2}}{2}\ \text{и}\ \frac{\sqrt{2}}{2}\), только с разными знаками.

α45°135°225°315°
π/43π/45π/47π/4
xcosα\(\frac{\sqrt{2}}{2}\)\(-\frac{\sqrt{2}}{2}\)\(-\frac{\sqrt{2}}{2}\)\(\frac{\sqrt{2}}{2}\)
ysinα\(\frac{\sqrt{2}}{2}\)\(\frac{\sqrt{2}}{2}\)\(-\frac{\sqrt{2}}{2}\)\(-\frac{\sqrt{2}}{2}\)

п.7. Синусы и косинусы углов π/6+πk/2

Базовыми точками на числовой окружности для угла 30° и всех отстоящих от него на углы, кратные прямому (углы \(\frac{\pi}{6}+\frac{\pi k}{2}\)), будут четыре точки: 30°, 120°, 210°, 300° \(\left(\frac{\pi}{6},\ \frac{2\pi}{3},\ \frac{7\pi}{6},\ \frac{5\pi}{3}\right)\). {\circ}}=\sqrt{1-\frac14}=\frac{\sqrt{3}}{2}. $$ Все исследуемые точки на числовой окружности будут иметь пару координат из чисел \(\frac12\ \text{и}\ \frac{\sqrt{3}}{2}\) в разном порядке и с разными знаками. Чтобы различать их «на глаз», заметим, что \(\frac{\sqrt{3}}{2}\approx 0,87\gt\frac12\). Т.е, отрезок короче будет равен по модулю \(\frac12\), а длиннее \(\frac{\sqrt{3}}{2}\).

α30°120°210°300°
π/62π/3 7π/65π/3
xcosα\(\frac{\sqrt{3}}{2}\)\(-\frac12\)\(-\frac{\sqrt{3}}{2}\)\(\frac12\)
ysinα\(\frac12\)\(\frac{\sqrt{3}}{2}\)\(-\frac12\)\(-\frac{\sqrt{3}}{2}\)

п.8. Синусы и косинусы углов π/3+πk/2

Базовыми точками на числовой окружности для угла 60° и всех отстоящих от него на углы, кратные прямому (углы \(\frac{\pi}{3}+\frac{\pi k}{2}\)), будут четыре точки: 60°, 150°, 240°, 330° \(\left(\frac{\pi}{3},\ \frac{5\pi}{6},\ \frac{4\pi}{3},\ \frac{11\pi}{6}\right)\). {\circ}}=\sqrt{1-\frac14}=\frac{\sqrt{3}}{2}. $$ Все исследуемые точки на числовой окружности будут иметь пару координат из чисел \(\frac12\ \text{и}\ \frac{\sqrt{3}}{2}\) в разном порядке и с разными знаками. Чтобы различать их «на глаз», заметим, что \(\frac{\sqrt{3}}{2}\approx 0,87\gt\frac12\). Т.е, отрезок короче будет равен по модулю \(\frac12\), а длиннее \(\frac{\sqrt{3}}{2}\).

α60°150°240°330°
π/35π/64π/311π/6
xcosα\(\frac12\)\(-\frac{\sqrt{3}}{2}\)\(-\frac12\)\(\frac{\sqrt{3}}{2}\)
ysinα\(\frac{\sqrt{3}}{2}\)\(\frac12\)\(-\frac{\sqrt{3}}{2}\)\(-\frac12\)

п.9. Примеры

Пример 1.
а) Найдите косинус угла α, если известно, что \(sin\alpha=0,8,\ \frac\pi2 \lt \alpha \lt \pi\)
Угол находится во второй четверти, значит, косинус отрицательный:

\(cos\alpha=-\sqrt{1-sin^2\alpha}=-\sqrt{1-0,8^2}=-\sqrt{0,36}=-0,6\)

б) Найдите синус угла, если известно, что \(cos\alpha=\frac{5}{13},\ -\frac\pi2 \lt \alpha \lt 0\)
Угол находится в четвертой четверти, значит синус отрицательный:
\(sin\alpha=-\sqrt{1-cos^2\alpha}=-\sqrt{1-\frac{5}{13}^2}=-\sqrt{\frac{144}{169}}=-\frac{12}{13}\)

Пример 2. Сравните числа
а) sin10° и sin320°
Угол 10° находится в 1-й четверти, sin⁡10° > 0
Угол 320° находится в 4-й четверти, sin⁡320° < 0
Получаем: sin⁡320° < 0 < sin10°
sin⁡10° > sin320°.

б) cos115° и sin85°
Угол 85° находится в 1-й четверти, sin85° > 0
Угол 115° находится во 2-й четверти, cos115° < 0
Получаем: cos115° < 0 < sin85°
cos115° < sin85°.

в) \(sin\frac{8\pi}{7}\) и \(cos\frac{11\pi}{25}\)
\(\pi\lt \frac{8\pi}{7}\lt \frac{3\pi}{2}\Rightarrow\) угол \(\frac{8\pi}{7}\) находится в 3-й четверти, \(sin\frac{8\pi}{7}\lt 0\)
\(0\lt \frac{11\pi}{25}\lt \frac{\pi}{2}\Rightarrow\) угол \(\frac{11\pi}{25}\) находится в 1-й четверти, \(cos\frac{11\pi}{25}\gt 0\)
Получаем: \(sin\frac{8\pi}{7} \lt 0\lt cos\frac{11\pi}{25}\)
\(sin\frac{8\pi}{7}\lt cos\frac{11\pi}{25}\)

Пример 3. Заданы точки на числовой окружности. Найдите их координаты

\begin{gather*} a)\ M\left(\frac{5\pi}{6}\right)\\ \frac{5\pi}{6}=5\cdot \frac\pi6=5\cdot30^{\circ}=150^{\circ} \end{gather*}
Косинус – длинный отрицательный \((-\frac{\sqrt{3}}{2})\)
Синус – короткий положительный \(\left(\frac12\right)\) \begin{gather*} x=cos\frac{5\pi}{6}=-\frac{\sqrt{3}}{2}\\ y=sin\frac{5\pi}{6}=\frac12 \end{gather*}
\begin{gather*} б)\ M\left(-\frac{2\pi}{3}\right)\\ -\frac{2\pi}{3}=-2\cdot \frac\pi3=-2\cdot 60^{\circ}=-120^{\circ} \end{gather*}
Косинус – короткий отрицательный \(\left(-\frac12\right)\)
Синус – длинный отрицательный \((-\frac{\sqrt{3}}{2})\) \begin{gather*} x=cos\left(-\frac{2\pi}{3}\right)=-\frac12\\ y=sin\left(-\frac{2\pi}{3}\right)=-\frac{\sqrt{3}}{2} \end{gather*}
\begin{gather*} в)\ M\left(\frac{5\pi}{2}\right)\\ \frac{5\pi}{2}=\left(2+\frac12\right)\pi=2\pi+\frac\pi2\rightarrow\frac\pi2=90^{\circ} \end{gather*} \begin{gather*} x=cos\frac{5\pi}{2}=cos\frac\pi2=0\\ y=sin\frac{5\pi}{2}=sin\frac\pi2=1 \end{gather*}\begin{gather*} г)\ M\left(-\frac{16\pi}{3}\right)\\ -\frac{16\pi}{3}=\frac{-18+2}{3}\pi=-6\pi+\frac{2\pi}{3}\rightarrow\frac{2\pi}{3}= 120^{\circ} \end{gather*}
Косинус – короткий отрицательный \(\left(-\frac12\right)\)
Синус – длинный положительный \(\left(\frac{\sqrt{3}}{2}\right)\) \begin{gather*} x=cos\left(-\frac{16\pi}{3}\right)=cos\frac{2\pi}{3}=-\frac12\\ y=sin\left(-\frac{16\pi}{3}\right)=sin\frac{2\pi}{3}=\frac{\sqrt{3}}{2} \end{gather*}

Пример 4. Найти sin⁡t,cos⁡t для данных t.

\(a)\ t=\frac{13\pi}{4}\)

Отнимем полный оборот: \(\frac{13\pi}{4}-2\pi=\frac{13-8}{4}\pi=\frac{5\pi}{4}\)
Угол кратный \(\frac\pi4\), его синус и косинус по модулю равны \(\frac{\sqrt{2}}{2}\), знаки определяются расположением угла.
\(\pi\lt\frac{5\pi}{4}\lt\frac{3\pi}{2}\Rightarrow\) угол находится в 3-й четверти, синус и косинус отрицательные.
Получаем: \begin{gather*} sin\frac{13\pi}{4}=sin\frac{5\pi}{4}=-\frac{\sqrt{2}}{2}\\ cos\frac{13\pi}{4}=cos\frac{5\pi}{4}=-\frac{\sqrt{2}}{2} \end{gather*}

\(б)\ t=\frac{11\pi}{2}\)

Отнимем 2 полных оборота: \(\frac{11\pi}{2}-2\cdot 2\pi=\frac{11-8}{2}\pi=\frac{3\pi}{2}\)
Угол кратный \(\frac\pi2\), находится на оси Y, в нижней точке числовой окружности.
Получаем: \begin{gather*} sin\frac{11\pi}{2}=sin\frac{3\pi}{2}=-1\\ cos\frac{11\pi}{2}=cos\frac{3\pi}{2}=0 \end{gather*}

\(в)\ t=\frac{17\pi}{6}\)

Отнимем полный оборот: \(\frac{17\pi}{6}-2\pi=\frac{17-12}{6}\pi=\frac{5\pi}{6}\)
Угол типа \(\frac\pi6\), в котором синус и косинус – это пара из \(\frac12\) и \(\frac{\sqrt{3}}{2}\), фактическое значение определяется по чертежу, исходя из того, что \(\frac{\sqrt{3}}{2}\approx 0,87\gt\frac12\). Знаки определяются по расположению угла в четверти: \(\frac\pi2\lt\frac{5\pi}{6}\lt\pi\Rightarrow\) угол находится во 2-й четверти.
Из чертежа получаем:
Косинус – длинный отрицательный
Синус – короткий положительный
Таким образом: \begin{gather*} sin\frac{17\pi}{6}=sin\frac{5\pi}{6}=\frac12\\ cos\frac{17\pi}{6}=cos\frac{5\pi}{6}=-\frac{\sqrt{3}}{2} \end{gather*}

\(г)\ t=-\frac{4\pi}{3}\)

Добавим полный оборот: \(-\frac{4\pi}{3}+2\pi=\frac{-4+6}{3}\pi=\frac{2\pi}{3}\)
Угол типа \(\frac\pi3\), в котором синус и косинус – это пара из \(\frac12\) и \(\frac{\sqrt{3}}{2}\), фактическое значение определяется по чертежу, исходя из того, что \(\frac{\sqrt{3}}{2}\approx 0,87\gt\frac12\). Знаки определяются по расположению угла в четверти: \(\frac\pi2\lt\frac{2\pi}{3}\lt\pi\Rightarrow\) угол находится во 2-й четверти.
Из чертежа получаем:
Косинус – короткий отрицательный
Синус – длинный положительный
Таким образом: \begin{gather*} sin\left(-\frac{4\pi}{3}\right)=sin\frac{2\pi}{3}=\frac{\sqrt{3}}{2}\\ cos\left(-\frac{4\pi}{3}\right)=cos\frac{2\pi}{3}=-\frac12 \end{gather*}

Синус и косинус на тригонометрической окружности

Главная / i / t

Косинус на тригонометрической окружности

В прошлый раз мы узнали, что из себя представляют такие тригонометрические функции как синус, косинус, тангенс и котангенс только для острых (от 0° до 90°) углов, но это значительно поможет тому, чтобы перейти к следующему, более точному, определению тригонометрической функции для любого угла от 0° до 360°.

Нарисуем обычную систему координат, только дополним еще ее окружностью, центр которой находится в точке A, а ее радиус равен единице. Затем проведем луч AB с началом в точке A, пересекающий окружность в точке B. Таким образом у нас получится тригонометрическая окружность (или единичная окружность).

Опустим перпендикуляр из точки B на ось X. И у нас опять образовался прямоугольный треугольник ABC, в котором сторона AB является не только гипотенузой, но и еще радиусом окружности.

Чтобы вычислить косинус угла, нам понадобится длину прилежащего к углу AC катета поделить на длину гипотенузы AB. Только, в отличие от предыдущего раза, длину катета AC в этот раз не обязательно нужно будет измерять, т.к. длина катета AC будет полностью совпадать с координатой точки C на оси X, поэтому достаточно просто посмотреть какая координата на оси X у точки C. Точка C будет как бы находиться уже на линейке, которой является ось X.

Попробуем определить косинус 60° уже с помощью тригонометрической окружности: по оси X видно, что координата точки C равна 0. 5, значит, и длина катета AC тоже равна 0.5. А длина гипотенузы AB является радиусом окружности и всегда (независимо от угла) будет равна 1. Остается только поделить длину AC (или координату точки C) на длину гипотенузы AB, равную 1:

Но при делении на единицу любое число не изменится, значит, можно было бы просто посмотреть на координату точки C на оси X – она и будет являться косинусом 60°. Именно по координате точки C и надо будет теперь всегда определять косинус угла.

A как же определить, чему равен косинус 120°? Невозможно же будет начертить прямоугольный треугольник с углом 120°, т.к. сумма углов в любом треугольнике всегда равна 180°, а в прямоугольном треугольнике с углом 120° сумма только двух углов уже больше 180°: 120°+90°=210°. На следующем рисунке изображена попытка нарисовать такой треугольник.

Зато угол 120° можно отмерить в тригонометрической окружности, опустить перпендикуляр BC из точки B на ось X, и тогда координата точки C на оси X будет являться косинусом 120°.

Точка C на оси X разместилась ровно на числе -0.5, значит, cos 120° = -0.5.

Полученное число -0,5 легко проверить с помощью обычного калькулятора, поставленного в инженерный режим, такой калькулятор легко найти в любом компьютере.

И калькулятор полностью согласен с нами.

Таким образом для приближенного определения косинуса угла α достаточно просто провести луч AB так, чтобы получился нужный угол, опустить перпендикуляр BC из точки B на ось X, и тогда точка C на оси X, как раз и укажет на число равное косинусу этого угла. Более того, теперь угол можно задать абсолютно любой от 0° до 360°.

Для примера попробуем приближенно определить косинус 210°. Отмерим угол 210° на окружности и опустим перпендикуляр BC из точки C на ось X. Точка C встанет где-то на числе -0,87, это значит, что косинус 210° будет приблизительно равен -0,87.

Калькулятор показал нам почти такой же результат (только более точный), следовательно, мы все правильно сделали. Aбсолютно точное же значение косинуса 210° равно , это число придется запомнить, но единичная окружность очень сильно облегчает этот процесс (тригонометрическая окружность с точными значениями синуса и косинуса всех основных углов будет приведена ниже).

Далее приведен рисунок с аналогичным примером косинуса 315° на тригонометрической окружности

Синус на тригонометрической окружности

С синусом дело обстоит очень похоже. Если пользоваться старым определением то, синус угла A получится, если длину противолежащей углу A стороны BC разделить на длину все той же гипотенузы AB, являющуюся радиусом единичной окружности. То есть деление стороны BC будет осуществляться тоже на единицу, а значит и синус угла A при любом угле всегда будет равен катету BC:

Для удобства угол A обозначим греческой буквой α (читается — «альфа»). Опустим перпендикуляр BD из точки B на ось Y. Легко можно заметить, что отрезок BC будет равен координате точки D на оси Y, тогда синус угла α будет равен не только длине BC и AD, но и координате точки D:

Координатой точки D будет гораздо удобнее пользоваться в будущем, нежели отрезком BC.

Получается, что при любом угле α sin α будет равен координате точки D на оси Y. Например, что синус 30° равен 0.5, гораздо легче заметить на тригонометрической окружности по координате точки D на оси Y.

A теперь попробуем приближенно определить синус 135°: отмерим на окружности угол 135°, проведем луч AB и опустим перпендикуляр BD из точки B на осьY, тогда точка D укажет на число, близкое к 0,71 на оси Y, значит, синус 135° приблизительно равен 0,71 (точно: sin 135° = ).

Точно таким же образом на следующих рисунках даны примеры определения sin 240°, sin 330° и сравнение этих результатов с калькулятором.

Обобщим сказанное:

Для определения синуса или косинуса угла нужно взять соответствующую этому углу точку на тригонометрической окружности, и тогда координата X этой точки будет равна косинусу этого угла, а координата Y будет равна синусу этого угла.

Точные синусы и косинусы всех основных (школьных) углов изображены на следующем рисунке.

Определим по этой окружности sin 300° и cos 300°. Находим на окружности точку 300° (зеленая точка), строим перпендикуляр (красная жирная линия) из этой точки на ось X, и он покажет на число 0.5, следовательно, cos 300°= 0.5. Строим перпендикуляр (синяя жирная линия) из зеленой точки на ось Y, и видим, что перпендикуляр покажет на число , значит sin 300° = .

Следует заметить, что на тригонометрической окружности есть некие особые углы такие как 90°, 180°, 270° и 360°. Если отмерить угол 90 и опустить перпендикуляр BC из точки B на ось X, то точка C сольется с точкой A и на оси окажется точно на нуле т.е. cos 90°=0.

Если точку B на тригонометрической окружности плавно перемещать от 0° до 90°, и при этом следить за координатой точки B, то будет заметно, что по мере приближения угла к 90° координата точки D на оси Y возрастает и приближается к 1. Значит, когда угол α станет равным 90°, то синус α будет равным 1, таким образом, sin 90°=1.

Рассуждая аналогичным образом, с помощью тригонометрической окружности можно легко установить следующее:

Для лучшего усвоения следует это проверить это самостоятельно.

Тригонометрический круг со всеми значениями, числовая окружность синус косинус тангенс котангенс, как пользоваться тригонометрическим кругом

Тригонометрический круг один из основных элементов геометрии для решения уравнений с синусом, косинусом, тангенсом и котангенсом.

Каково определение данного термина, как строить данный круг, как определить четверть в тригонометрии, как узнать углы в построенном тригонометрическом круге — об этом и многом другом расскажем далее.

Содержание

Тригонометрическая окружность

Тригонометрическим видом числовой окружности в математике является круг, имеющий одинарный радиус с центром в начале координатной плоскости. Как правило, она образована пространством из формул синуса с косинусом, тангенсом и котангенсом на системе координат.

Назначение такой сферы с n-мерным пространством в том, что благодаря ей могут быть описаны тригонометрические функции. Выглядит она просто: круг, внутри которого находится система координат и множественные прямоугольного вида треугольники, образованные из этой окружности по тригонометрическим функциям.

Что такое синус, косинус, тангенс, котангенс в прямоугольном треугольнике

Прямоугольный вид треугольника — это тот, у которого один из углов равен 90°. Он образован катетами и гипотенузой со всеми значениями тригонометрии. Катеты две стороны треугольника, которые прилегают к углу 90°, а третья гипотенуза, она всегда длиннее катетов.

Синусом называется отношение одного из катетов к гипотенузе, косинусом отношение другого катета к ней, а тангенсом отношение двух катетов. Отношение символизирует деление. Также тангенсом является деление острого угла на синус с косинусом. Котангенсом является противоположное тангенсу отношение.

Формулы последних двух отношений выглядят следующим образом: tg(a) = sin(a) / cos(a) и ctg(a) = cos(a) / sin(a).

Построение единичной окружности

Построение единичной окружности сводится к ее прорисовке с единичным радиусом в центре системы координат. Затем для построения нужно отсчитать углы и, двигаясь против часовой стрелки, обойти по целому кругу, проставляя соответствующие им линии координаты.

Начинается построение после черчения круга и установки точки в его центре с размещения системы координат ОХ. Точкой О сверху оси координат является синус, а Х косинус. Соответственно они являются абсциссой и ординатой. Затем нужно провести измерения ∠. Они проводятся градусами и радианами.

Сделать перевод этих показателей просто полный круг равен двум пи радиан. Угол от нуля против часовой стрелки идет со знаком +, а ∠ от 0 по часовой стрелке со знаком -. Положительные и отрицательные значения синуса с косинусом повторяются каждый оборот круга.

Углы на тригонометрическом круге

Для того, чтобы освоить теорию тригонометрической окружности, нужно понять, как считаются ∠ на ней, и в чем они измеряются. Считаются они очень просто.

Окружность делится системой координат на четыре части. Каждая часть образует ∠ 90°. Половина от этих углов равняется 45 градусам. Соответственно две доли окружности равняются 180°, а три 360°. Как пользоваться этой информацией?

Если требуется решить задачу по нахождению ∠, прибегают к теоремам о треугольниках и основным Пифагоровым законам, связанных с ними.

Измеряются углы в радианах:

  • от 0 до 90° значения углов от 0 до ∏/2,
  • от 90 до 180° значения углов от ∏/2 до ∏,
  • от 180 до 270° от ∏ до 3*∏/2,
  • последняя четверть от 2700 до 3600 — значения от 3*∏/2 до 2*∏.

Чтобы узнать конкретное измерение, перевести радианы в градусы или наоборот, следует прибегнуть к таблице-шпаргалке.

Перевод углов из градусов в радианы

Углы возможно измерить в градусах либо радианах. Требуется осознавать связь между обоими значениями. Эта взаимосвязь выражена в тригонометрии с помощью специальной формулы. Благодаря пониманию связи, можно научиться оперативным образом управлять углами и переходить от градусов к радианам обратно.

Для того чтобы точно узнать, чему равен один радиан, можно воспользоваться следующей формулой:

1 рад. = 180 / ∏ = 180 / 3,1416 = 57,2956

В конечном итоге, 1 радиан равен 57°, а в 1 градусе 0,0175 радиан:

1 градус = (∏ /180) рад. = 3,1416 / 180 рад. = 0,0175 рад.

Косинус, синус, тангенс, котангенс на тригонометрической окружности

Косинус с синусом, тангенсом и котангенсом на тригонометрической окружности функции углов альфа от 0 до 360 градусов. Каждая функция обладает положительным или отрицательным значением в зависимости от того, какая величина у угла. Они символизируют отношения к прямоугольным треугольникам, образованным в круге.

Заключение

В целом, тригонометрическая окружность – единичная окружность, необходимая для решения соответствующих задач и описания функций. Она состоит из многих составляющих, запомнить которые нужно обязательно для правильного решения последующих задач.

Mathway | Популярные задачи

1Найти точное значениеsin(30)
2Найти точное значениеsin(45)
3Найти точное значениеsin(30 град. )
4Найти точное значениеsin(60 град. )
5Найти точное значениеtan(30 град. )
6Найти точное значениеarcsin(-1)
7Найти точное значениеsin(pi/6)
8Найти точное значениеcos(pi/4)
9Найти точное значениеsin(45 град. )
10Найти точное значениеsin(pi/3)
11Найти точное значениеarctan(-1)
12Найти точное значениеcos(45 град. )
13Найти точное значениеcos(30 град. )
14Найти точное значениеtan(60)
15Найти точное значениеcsc(45 град. )
16Найти точное значениеtan(60 град. )
17Найти точное значениеsec(30 град. )
18Найти точное значениеcos(60 град. )
19Найти точное значениеcos(150)
20Найти точное значениеsin(60)
21Найти точное значениеcos(pi/2)
22Найти точное значениеtan(45 град. )
23Найти точное значениеarctan(- квадратный корень из 3)
24Найти точное значениеcsc(60 град. )
25Найти точное значениеsec(45 град. )
26Найти точное значениеcsc(30 град. )
27Найти точное значениеsin(0)
28Найти точное значениеsin(120)
29Найти точное значениеcos(90)
30Преобразовать из радианов в градусыpi/3
31Найти точное значениеtan(30)
32Преобразовать из градусов в радианы45
33Найти точное значениеcos(45)
34Упроститьsin(theta)^2+cos(theta)^2
35Преобразовать из радианов в градусыpi/6
36Найти точное значениеcot(30 град. )
37Найти точное значениеarccos(-1)
38Найти точное значениеarctan(0)
39Найти точное значениеcot(60 град. )
40Преобразовать из градусов в радианы30
41Преобразовать из радианов в градусы(2pi)/3
42Найти точное значениеsin((5pi)/3)
43Найти точное значениеsin((3pi)/4)
44Найти точное значениеtan(pi/2)
45Найти точное значениеsin(300)
46Найти точное значениеcos(30)
47Найти точное значениеcos(60)
48Найти точное значениеcos(0)
49Найти точное значениеcos(135)
50Найти точное значениеcos((5pi)/3)
51Найти точное значениеcos(210)
52Найти точное значениеsec(60 град. )
53Найти точное значениеsin(300 град. )
54Преобразовать из градусов в радианы135
55Преобразовать из градусов в радианы150
56Преобразовать из радианов в градусы(5pi)/6
57Преобразовать из радианов в градусы(5pi)/3
58Преобразовать из градусов в радианы89 град.
59Преобразовать из градусов в радианы60
60Найти точное значениеsin(135 град. )
61Найти точное значениеsin(150)
62Найти точное значениеsin(240 град. )
63Найти точное значениеcot(45 град. )
64Преобразовать из радианов в градусы(5pi)/4
65Найти точное значениеsin(225)
66Найти точное значениеsin(240)
67Найти точное значениеcos(150 град. )
68Найти точное значениеtan(45)
69Вычислитьsin(30 град. )
70Найти точное значениеsec(0)
71Найти точное значениеcos((5pi)/6)
72Найти точное значениеcsc(30)
73Найти точное значениеarcsin(( квадратный корень из 2)/2)
74Найти точное значениеtan((5pi)/3)
75Найти точное значениеtan(0)
76Вычислитьsin(60 град. )
77Найти точное значениеarctan(-( квадратный корень из 3)/3)
78Преобразовать из радианов в градусы(3pi)/4
79Найти точное значениеsin((7pi)/4)
80Найти точное значениеarcsin(-1/2)
81Найти точное значениеsin((4pi)/3)
82Найти точное значениеcsc(45)
83Упроститьarctan( квадратный корень из 3)
84Найти точное значениеsin(135)
85Найти точное значениеsin(105)
86Найти точное значениеsin(150 град. )
87Найти точное значениеsin((2pi)/3)
88Найти точное значениеtan((2pi)/3)
89Преобразовать из радианов в градусыpi/4
90Найти точное значениеsin(pi/2)
91Найти точное значениеsec(45)
92Найти точное значениеcos((5pi)/4)
93Найти точное значениеcos((7pi)/6)
94Найти точное значениеarcsin(0)
95Найти точное значениеsin(120 град. )
96Найти точное значениеtan((7pi)/6)
97Найти точное значениеcos(270)
98Найти точное значениеsin((7pi)/6)
99Найти точное значениеarcsin(-( квадратный корень из 2)/2)
100Преобразовать из градусов в радианы88 град.

Тригонометрическая окружность. Исчерпывающее руководство (2019)

Видеоурок «Определение синуса и косинуса на единичной окружности» представляет наглядный материал для урока по соответствующей теме. В ходе урока рассматриваются понятия синуса и косинуса для чисел, соответствующих точкам единичной окружности, описывается множество примеров, формирующих умение решать задания, где используется данная интерпретация понятий. Удобное и понятное иллюстрирований решений, подробно описанный ход рассуждений помогают быстрее достичь целей обучения, повысить эффективность урока.

Видеоурок начинается с представления темы. В начале демонстрации дается определение синуса и косинуса числа. На экране демонстрируется единичная окружность с центром в начале координат, отмечаются точки пересечения единичной окружности с осями координат А, В, С, D. В рамке выделено определение, в котором указано, что если точке М, принадлежащей единичной окружности, соответствует некоторое число t, то абсцисса этой точки является косинусом числа t и обозначается cos t, ордината точки является синусом и обозначается sin t. Озвучивание определения сопровождается изображением на единичной окружности точки М, указанием ее абсциссы и ординаты. Представляется краткая запись с помощью обозначений, что для М(t)=M(x;y), х= cos t, у= sin t. Указываются ограничения, накладываемые на значение косинуса и синуса числа. Согласно рассмотренным данным, -1

Также по рисунку легко отследить, как изменяется знак функции в зависимости от того, в какой четверти располагается точка. На экране составляется таблица, в которой для каждой функции указывается ее знак в зависимости от четверти. Знак cos t — плюс в первой и четвертой четвертях и минус во второй и третьей четвертях. Знак sin t — плюс в первой и второй четвертях, минус в третьей и четвертой четвертях.

Ученикам напоминается уравнение единичной окружности х 2 +у 2 =1. Отмечается, что после подстановки вместо координат соответствующих функций, получим cos 2 t+ sin 2 t=1 — основное тригонометрическое тождество. Пользуясь способом нахождения sin t и cos t с помощью единичной окружности, заполняется таблица основных значений синуса и косинуса для чисел от 0 до 2π с шагом π/4 и для чисел от π/6 до 11π/6 с шагом π/6. На экране демонстрируются эти таблицы. С помощью их и рисунка учитель может проверить, как усвоен материал и насколько ученикам понятно происхождение значений sin t и cos t.

Рассматривается пример, в котором вычисляется sin t и cos t для t=41π/4. Решение иллюстрируется рисунком, на котором изображена единичная окружность с центром в начале координат. На ней отмечается точка 41π/4. Замечено, что данная точка совпадает с положением точки π/4. Это доказывается с помощью представления данной дроби в виде смешанной 41π/4=π/4+2π·5. Пользуясь таблицей значений косинуса, получаем значения cos π/4=√2/2 и sinπ/4=√2/2. Из полученных сведений следует, что cos 41π/4=√2/2 и sin 41π/4=√2/2.

В втором примере необходимо вычислить sin t и cos t для t=-25π/3. На экране изображается единичная окружность с отмеченной на ней точкой t=-25π/3. Сначала для решения задания число -25π/3 представляется в виде смешанной дроби, чтобы обнаружить, какому табличному значению будет соответствовать его sin t и cos t. После преобразования получаем -25π/3=-π/3+2π·(-4). Очевидно, t=-25π/3 совпадет на окружности с точкой -π/3 или 5π/3. Из таблицы выбираем соответствующие значения синуса и косинуса cos 5π/3=1/2 и sin 5π/3=-√3/2. Эти значения будут верными и для рассматриваемого числа cos (-25π/3)=1/2 и sin (-25π/3)=-√3/2. Задача решена.

Аналогично решается и пример 3, в котором необходимо вычислить sin t и cos t для t=37π. Чтобы решить пример, число 37π раскладывается, вычленяя π и 2π. В таком представлении получается 37π=π+2π·18. На единичной окружности, которая изображена рядом с решением, отмечается данная точка на пересечении отрицательной части оси ординат и единичной окружности — точка π. Очевидно, что значения синуса и косинуса числа совпадут с табличными значениями π. Из таблицы находим значения sin π=-1 и cos π=0. Соответственно, эти же значения являются искомыми, то есть sin 37π=-1 и cos 37π=0.

В примере 4 требуется вычислить sin t и cos t при t=-12π. Представляем число в виде -12π=0+2π·(-6). Соответственно, точка -12π совпадает с точкой 0. Значения косинуса и синуса этой точки sin 0=1 и cos 0=0. Эти значения и являются искомыми sin (-12π)=1 и cos (-12π)=0.

В пятом примере нужно решить уравнение sin t=√3/2. В решении уравнения используется понятие синуса числа. Так как он представляет ординату точки М(t), то необходимо отыскать точку с ординатой √3/2. На рисунке, сопровождающем решение, видно, что ординате √3/2 соответствуют две точки — первая π/3 и вторая 2π/3. Учитывая периодичность функции, отмечаем, что t=π/3+2πk и t= 2π/3+2πk для целого k.

В примере 6 решается уравнение с косинусом — cos t=-1/2. В поиске решений уравнения находим на единичной окружности точки с абсциссой 2π/3. На экране демонстрируется рисунок, на котором отмечается абсцисса -1/2. Ей соответствуют две точки на окружности — 2π/3 и -2π/3. Учитывая периодичность функций, найденное решение записывается в виде t=2π/3+2πk и t=-2π/3+2πk, где k- целое число.

В примере 7 решается уравнение sin t-1=0. Чтобы найти решение, уравнение преобразуется к виду sin t=1. Синусу 1 соответствует число π/2. Учитывая периодичность функции, найденное решение записывается в виде t=π/2+2πk, где k — целое. Аналогично в примере 8 решается уравнение cos t+1=0. Преобразуем уравнение к виду cos t=-1. Точка, абсцисса которой равна -1, соответствует числу π. Эта точка отмечена на единичной окружности, изображенной рядом с текстовым решением. Соответственно, решением данного уравнения является число t=π+2πk, где k — целое число. Не более сложным является решение уравнения cos t+1=1 в примере 9. Преобразовав уравнение, получаем cos t=0. На единичной окружности, изображенной рядом с решением, отмечаем точки -π/2 и -3π/2, в которых косинус принимает значение 0. Очевидно, решением данного уравнение будет ряд значений t=π/2+πk, где k — целое число.

В примере 10 сравниваются значения sin 2 и cos 3. Чтобы решение было наглядным, демонстрируется рисунок, где отмечены точки 2 и 3. Зная, что π/2≈1,57, оцениваем удаленность точек от нее. На рисунке отмечается, что точка 2 удалена от π/2 на 0,43, в то время как 3 удалена на 1,43, поэтому точка 2 имеет большую абсциссу, чем точка 3. Это значит, что sin 2>cos 3.

Пример 11 описывает вычисление выражения sin 5π/4. Так как 5π/4 — это π/4+π, то, используя формулы приведения, выражение можно преобразовать в вид — sin π/4. Из таблицы выбираем его значение — sin π/4=-√2/2. Аналогично в примере 12 находится значение выражения cos7π/6. Преобразуя его к виду cos(π/6+π), получаем выражение — cos π/6. Табличное значение — cos π/6=-√3/2. Это значение и будет решением.

Далее предлагается запомнить важные равенства, которые помогают в решении задач — это sin(-t)= -sin t и cos (-t)=cos t. Фактически данное выражение отображает четность косинуса и нечетность синуса. На изображении единичной окружности рядом с равенствами можно увидеть, как на координатной плоскости работают данные равенства. Также представляются два равенства, отображающие периодичность функций, важные для решения задач sin(t+2πk)= sin t и cos (t+2πk)=cos t. Демонстрируются равенства, отображающие симметричное расположение точек на единичной окружности sin(t+π)= -sin t и cos (t+π)=-cos t. Рядом с равенствами строится изоражение, на котором отображается расположение этих точек на единичной окружности. И последние представленные равенства sin(t+π/2)= cos t и cos (t+π/2)=- sin t.

Видеоурок «Определение синуса и косинуса на единичной окружности» рекомендуется применять на традиционном школьном уроке математик для повышения его эффективности, обеспечения наглядности объяснения учителя. С этой же целью материал может использоваться в ходе дистанционного обучения. Пособие также может быть полезно для формирования соответствующих навыков решения заданий у учеников при самостоятельном освоении материала.

ТЕКСТОВАЯ РАСШИФРОВКА:

«Определение синуса и косинуса на единичной окружности».

Дадим определение синуса и косинуса числа

ОПРЕДЕЛЕНИЕ: если точка М числовой единичной окружности соответствует числу t(тэ), то абсциссу точки М называют косинусом числа t(тэ) и обозначают cost, а ординату точки М называют синусом числа t(тэ) и обозначают sint(рис).

Значит, если М(t) = М (x ,y)(эм от тэ равно эм с координатами икс и игрек), то x = cost, y= sint (икс равен косинус тэ, игрек равен синус тэ).Следовательно, -1≤ cost ≤ 1, -1≤ sint ≤1(косинус тэ больше либо равно минус один, но меньше либо равно один; синус тэ больше либо равно минус один, но меньше либо равно один).Зная, что каждая точка числовой окружности имеет в системе xOy свои координаты, можно составить таблицу значении синуса и косинуса по четвертям окружности, где значение косинуса положительно в первой и четвертой четвертях и, соответственно, отрицательно во второй и третьей четвертях.

Значение синуса положительно в первой и второй четвертях и, соответственно, отрицательно в третьей и четвертой четвертях. (показать на чертеже)

Так как уравнение числовой окружности имеет вид х 2 + у 2 = 1(икс квадрат плюс игрек квадрат равно одному), то получаем равенство:

(косинус квадрат тэ плюс синус квадрат тэ равно единице).

Опираясь на таблицы, которые мы составляли при определении координат точек числовой окружности, составим таблицы для координат точек числовой окружности для значений cost и sint .

Рассмотрим примеры.

ПРИМЕР 1. Вычислить cos t и sin t, если t = (тэ равно сорок один пи на четыре).

Решение. Числу t = соответствует та же точка числовой окружности, что и числу, так как = ∙π = (10 +) ∙π = + 2π ∙ 5(сорок один пи на четыре равно сумме пи на четыре и произведения два пи на пять). А для точки t = по таблице значение косинусов 1 имеем cos = и sin =. Следовательно,

ПРИМЕР 2. Вычислить cos t и sin t, если t = (тэ равно минус двадцать пять пи на три).

РЕШЕНИЕ: Числу t = соответствует та же точка числовой окружности, что и числу, так как = ∙ π = — (8 +)∙π = + 2π ∙ (- 4) (минус двадцать пять пи на три равно сумме минус пи на три и произведению двух пи на минус четыре). А числу соответствует на числовой окружности та же точка, что и числу. А для точки t = по таблице 2 имеем cos = и sin = .Следовательно, cos () = и sin () =.

ПРИМЕР 3. Вычислить cos t и sin t, если t = 37π; (тэ равно тридцать семь пи).

РЕШЕНИЕ: 37π = 36π + π = π + 2π ∙ 18.Значит, числу 37π соответствует та же точка числовой окружности, что и числу π. А для точки t = π по таблице 1 имеем cos π = -1, sin π=0.Значит, cos37π = -1, sin37π=0.

ПРИМЕР 4. Вычислить cos t и sin t, если t = -12π (равно минус двенадцать пи).

РЕШЕНИЕ: — 12π = 0 + 2π ∙ (- 6), то есть числу — 12π соответствует та же точка числовой окружности, что и числу ноль. А для точки t = 0 по таблице 1 имеем cos 0 = 1, sin 0 =0.Значит, cos(-12π) =1, sin(-12π) =0.

ПРИМЕР 5. Решить уравнение sin t = .

Решение. Учитывая, что sin t — это ордината точки М(t) (эм от тэ) числовой окружности, найдем на числовой окружности точки с ординатой и запишем каким числам t они соответствуют. Одна точка соответствует числу, а значит, и любому числу вида + 2πk. Вторая точка соответствует числу, а значит, и любому числу вида + 2πk. Ответ: t = + 2πk,где kϵZ (ка принадлежит зэт),t = + 2πk,где kϵZ (ка принадлежит зэт).

ПРИМЕР 6. Решить уравнение cos t = .

Решение. Учитывая, что cos t — это абсцисса точки М(t) (эм от тэ) числовой окружности, найдем на числовой окружности точки с абсциссой и запишем каким числам t они соответствуют. Одна точка соответствует числу,а значит и любому числу вида + 2πk. А вторая точка соответствует числу или, а значит, и любому числу вида + 2πk или + 2πk.

Ответ: t = + 2πk, t=+ 2πk (или ± + 2πk(плюс минус два пи на три плюс два пи ка) , где kϵZ (ка принадлежит зэт).

ПРИМЕР 7.Решить уравнение cos t = .

Решение. Аналогично предыдущему примеру, на числовой окружности нужно найти точки c абсциссой и записать, каким числам t они соответствуют.

По рисунку видно, что абсциссу имеют две точки Е и S, а каким числам они соответствуют мы пока не сможем сказать. К этому вопросу вернемся позже.

ПРИМЕР 8.Решить уравнение sin t = — 0,3.

Решение. На числовой окружности найдем точки с ординатой — 0,3 и запишем, каким числам t они соответствуют.

Ординату — 0,3 имеют две точки P и H, а каким числам они соответствуют мы пока не сможем сказать. К этому вопросу так же вернемся позже.

ПРИМЕР 9.Решить уравнение sin t -1 =0

Решение. Перенесем минус единицу в правую часть уравнения, получим синус тэ равно одному (sin t =1). На числовой окружности нам нужно найти точку, у которой ордината равна один. Эта точка соответствует числу, а значит всем числам вида + 2πk(пи на два плюс два пи ка).

Ответ: t = + 2πk, kϵZ(ка принадлежит зэт).

ПРИМЕР 10.Решить уравнение cos t + 1 = 0.

Перенесем единицу в правую часть уравнения, получим косинус тэ равно минус один(cos t = — 1).Абсциссу минус один имеет точка числовой окружности, которая соответствует числу π, а это значит, и все числам вида π+2πk. Ответ: t = π+ 2πk, kϵZ.

ПРИМЕР 11. Решить уравнение cos t + 1 = 1.

Перенесем единицу в правую часть уравнения, получим косинус тэ равно нулю(cos t = 0).Абсциссу ноль имеют точки В и D (рис 1), которые соответствуют числам, и т. д. Эти числа можно записать так + πk. Ответ: t = + πk, kϵZ.

ПРИМЕР 12. Какое из двух чисел больше, cos 2 или cos 3? (косинус двух или косинус трех)

Решение. Переформулируем вопрос по-другому: на числовой окружности отмечены точки 2 и 3. У какой из них абсцисса больше?

На числовой окружности отметим точки 2 и 3. Вспомним, что.Значит, точка 2 удалена от по окружности примерно на 0,43(нуль целых сорок три сотых) (2 -≈ 2 — 1,57 = 0,43), а точка 3 на 1,43 (одну целую сорок три сотых). Следовательно, точка 2 находится ближе к точке, чем точка 3, поэтому у нее абсцисса больше (мы учли, что абсциссы обе отрицательные).

Ответ: cos 2 > cos 3.

ПРИМЕР 13. Вычислить sin (синус пять пи на четыре)

Решение. sin(+ π) = — sin = (синус пять пи на четыре равно сумме пи на четыре и пи равно минус синус пи на четыре равно минус корень из двух на два).

ПРИМЕР 14. Вычислить cos (косинус семь пи на шесть).

cos(+ π) = — cos =. (представили семь пи на шесть как сумму пи на шесть и пи и применили третье равенство).

Для синуса и косинуса получим некоторые важные формулы.

1. Для любого значения t справедливы равенства

sin (-t) = -sin t

cos (-t) = cos t

Синус от минус тэ равно минус синус тэ

Косинус от мину тэ равно косинусу тэ.

По рисунку видно, что у точек Е и L, симметричных относительно оси абсцисс, одна и та же абсцисса, это значит

cos(-t) = cost, но равны по модулю и противоположные по знаку ординаты (это значит sin(- t) = — sint.

2. Для любого значения t справедливы равенства

sin (t+2πk) = sin t

cos (t+2πk) = cos t

Синус от тэ плюс два пи ка равно синусу тэ

Косинус от тэ плюс два пи ка равно косинусу тэ

Это верно, так как числам t и t+2πk соответствует одна и та же точка.

3. Для любого значения t справедливы равенства

sin (t+π) = -sin t

cos (t+π) = -cos t

Синус от тэ плюс пи равно минус синусу тэ

косинус от тэ плюс пи равно минус косинусу тэ

Пусть числу t соответствует точка E числовой окружности, тогда числу t+π соответствует точка L, которая симметрична точке E относительно начала координат. По рисунку видно, что у этих точек абсциссы и ординаты равны по модулю и противоположны по знаку. Это значит,

cos(t +π)= — cost;

sin(t +π)= — sint.

4. Для любого значения t справедливы равенства

sin (t+) = cos t

cos (t+) = -sin t

Синус тэ плюс пи на два равно косинусу тэ

Косинус тэ плюс пи на два равно минус синусу тэ.

В пятом веке до нашей эры древнегреческий философ Зенон Элейский сформулировал свои знаменитые апории, самой известной из которых является апория «Ахиллес и черепаха». Вот как она звучит:

Допустим, Ахиллес бежит в десять раз быстрее, чем черепаха, и находится позади неё на расстоянии в тысячу шагов. За то время, за которое Ахиллес пробежит это расстояние, черепаха в ту же сторону проползёт сто шагов. Когда Ахиллес пробежит сто шагов, черепаха проползёт ещё десять шагов, и так далее. Процесс будет продолжаться до бесконечности, Ахиллес так никогда и не догонит черепаху.

Это рассуждение стало логическим шоком для всех последующих поколений. Аристотель, Диоген, Кант, Гегель, Гильберт… Все они так или иначе рассматривали апории Зенона. Шок оказался настолько сильным, что «… дискуссии продолжаются и в настоящее время, прийти к общему мнению о сущности парадоксов научному сообществу пока не удалось… к исследованию вопроса привлекались математический анализ, теория множеств, новые физические и философские подходы; ни один из них не стал общепризнанным решением вопроса… » [Википедия, » Апории Зенона «]. Все понимают, что их дурят, но никто не понимает, в чем заключается обман.

С точки зрения математики, Зенон в своей апории наглядно продемонстрировал переход от величины к . Этот переход подразумевает применение вместо постоянных. Насколько я понимаю, математический аппарат применения переменных единиц измерения либо ещё не разработан, либо его не применяли к апории Зенона. Применение же нашей обычной логики приводит нас в ловушку. Мы, по инерции мышления, применяем постоянные единицы измерения времени к обратной величине. С физической точки зрения это выглядит, как замедление времени до его полной остановки в момент, когда Ахиллес поравняется с черепахой. Если время останавливается, Ахиллес уже не может перегнать черепаху.

Если перевернуть привычную нам логику, всё становится на свои места. Ахиллес бежит с постоянной скоростью. Каждый последующий отрезок его пути в десять раз короче предыдущего. Соответственно, и время, затрачиваемое на его преодоление, в десять раз меньше предыдущего. Если применять понятие «бесконечность» в этой ситуации, то правильно будет говорить «Ахиллес бесконечно быстро догонит черепаху».

Как избежать этой логической ловушки? Оставаться в постоянных единицах измерения времени и не переходить к обратным величинам. На языке Зенона это выглядит так:

За то время, за которое Ахиллес пробежит тысячу шагов, черепаха в ту же сторону проползёт сто шагов. За следующий интервал времени, равный первому, Ахиллес пробежит ещё тысячу шагов, а черепаха проползет сто шагов. Теперь Ахиллес на восемьсот шагов опережает черепаху.

Этот подход адекватно описывает реальность без всяких логических парадоксов. Но это не полное решение проблемы. На Зеноновскую апорию «Ахиллес и черепаха» очень похоже утверждение Эйнштейна о непреодолимости скорости света. Эту проблему нам ещё предстоит изучить, переосмыслить и решить. И решение нужно искать не в бесконечно больших числах, а в единицах измерения.

Другая интересная апория Зенона повествует о летящей стреле:

Летящая стрела неподвижна, так как в каждый момент времени она покоится, а поскольку она покоится в каждый момент времени, то она покоится всегда.

В этой апории логический парадокс преодолевается очень просто — достаточно уточнить, что в каждый момент времени летящая стрела покоится в разных точках пространства, что, собственно, и является движением. Здесь нужно отметить другой момент. По одной фотографии автомобиля на дороге невозможно определить ни факт его движения, ни расстояние до него. Для определения факта движения автомобиля нужны две фотографии, сделанные из одной точки в разные моменты времени, но по ним нельзя определить расстояние. Для определения расстояния до автомобиля нужны две фотографии, сделанные из разных точек пространства в один момент времени, но по ним нельзя определить факт движения (естественно, ещё нужны дополнительные данные для расчетов, тригонометрия вам в помощь). На что я хочу обратить особое внимание, так это на то, что две точки во времени и две точки в пространстве — это разные вещи, которые не стоит путать, ведь они предоставляют разные возможности для исследования.

среда, 4 июля 2018 г.

Очень хорошо различия между множеством и мультимножеством описаны в Википедии . Смотрим.

Как видите, «во множестве не может быть двух идентичных элементов», но если идентичные элементы во множестве есть, такое множество называется «мультимножество». Подобную логику абсурда разумным существам не понять никогда. Это уровень говорящих попугаев и дрессированных обезьян, у которых разум отсутствует от слова «совсем». Математики выступают в роли обычных дрессировщиков, проповедуя нам свои абсурдные идеи.

Когда-то инженеры, построившие мост, во время испытаний моста находились в лодке под мостом. Если мост обрушивался, бездарный инженер погибал под обломками своего творения. Если мост выдерживал нагрузку, талантливый инженер строил другие мосты.

Как бы математики не прятались за фразой «чур, я в домике», точнее «математика изучает абстрактные понятия», есть одна пуповина, которая неразрывно связывает их с реальностью. Этой пуповиной являются деньги. Применим математическую теорию множеств к самим математикам.

Мы очень хорошо учили математику и сейчас сидим в кассе, выдаем зарплату. Вот приходит к нам математик за своими деньгами. Отсчитываем ему всю сумму и раскладываем у себя на столе на разные стопки, в которые складываем купюры одного достоинства. Затем берем с каждой стопки по одной купюре и вручаем математику его «математическое множество зарплаты». Поясняем математику, что остальные купюры он получит только тогда, когда докажет, что множество без одинаковых элементов не равно множеству с одинаковыми элементами. Вот здесь начнется самое интересное.

В первую очередь, сработает логика депутатов: «к другим это применять можно, ко мне — низьзя!». Дальше начнутся уверения нас в том, что на купюрах одинакового достоинства имеются разные номера купюр, а значит их нельзя считать одинаковыми элементами. Хорошо, отсчитываем зарплату монетами — на монетах нет номеров. Здесь математик начнет судорожно вспоминать физику: на разных монетах имеется разное количество грязи, кристаллическая структура и расположение атомов у каждой монеты уникально…

А теперь у меня самый интересный вопрос: где проходит та грань, за которой элементы мультимножества превращаются в элементы множества и наоборот? Такой грани не существует — всё решают шаманы, наука здесь и близко не валялась.

Вот смотрите. Мы отбираем футбольные стадионы с одинаковой площадью поля. Площадь полей одинакова — значит у нас получилось мультимножество. Но если рассматривать названия этих же стадионов — у нас получается множество, ведь названия разные. Как видите, один и тот же набор элементов одновременно является и множеством, и мультимножеством. Как правильно? А вот здесь математик-шаман-шуллер достает из рукава козырный туз и начинает нам рассказывать либо о множестве, либо о мультимножестве. В любом случае он убедит нас в своей правоте.

Чтобы понять, как современные шаманы оперируют теорией множеств, привязывая её к реальности, достаточно ответить на один вопрос: чем элементы одного множества отличаются от элементов другого множества? Я вам покажу, без всяких «мыслимое как не единое целое» или «не мыслимое как единое целое».

воскресенье, 18 марта 2018 г.

Сумма цифр числа — это пляска шаманов с бубном, которая к математике никакого отношения не имеет. Да, на уроках математики нас учат находить сумму цифр числа и пользоваться нею, но на то они и шаманы, чтобы обучать потомков своим навыкам и премудростям, иначе шаманы просто вымрут.

Вам нужны доказательства? Откройте Википедию и попробуйте найти страницу «Сумма цифр числа». Её не существует. Нет в математике формулы, по которой можно найти сумму цифр любого числа. Ведь цифры — это графические символы, при помощи которых мы записываем числа и на языке математики задача звучит так: «Найти сумму графических символов, изображающих любое число». Математики эту задачу решить не могут, а вот шаманы — элементарно.

Давайте разберемся, что и как мы делаем для того, чтобы найти сумму цифр заданного числа. И так, пусть у нас есть число 12345. Что нужно сделать для того, чтобы найти сумму цифр этого числа? Рассмотрим все шаги по порядку.

1. Записываем число на бумажке. Что же мы сделали? Мы преобразовали число в графический символ числа. Это не математическое действие.

2. Разрезаем одну полученную картинку на несколько картинок, содержащих отдельные цифры. Разрезание картинки — это не математическое действие.

3. Преобразовываем отдельные графические символы в числа. Это не математическое действие.

4. Складываем полученные числа. Вот это уже математика.

Сумма цифр числа 12345 равна 15. Вот такие вот «курсы кройки и шитья» от шаманов применяют математики. Но это ещё не всё.

С точки зрения математики не имеет значения, в какой системе счисления мы записываем число. Так вот, в разных системах счисления сумма цифр одного и того же числа будет разной. В математике система счисления указывается в виде нижнего индекса справа от числа. С большим числом 12345 я не хочу голову морочить, рассмотрим число 26 из статьи про . Запишем это число в двоичной, восьмеричной, десятичной и шестнадцатеричной системах счисления. Мы не будем рассматривать каждый шаг под микроскопом, это мы уже сделали. Посмотрим на результат.

Как видите, в разных системах счисления сумма цифр одного и того же числа получается разной. Подобный результат к математике никакого отношения не имеет. Это всё равно, что при определении площади прямоугольника в метрах и сантиметрах вы получали бы совершенно разные результаты.

Ноль во всех системах счисления выглядит одинаково и суммы цифр не имеет. Это ещё один аргумент в пользу того, что . Вопрос к математикам: как в математике обозначается то, что не является числом? Что, для математиков ничего, кроме чисел, не существует? Для шаманов я могу такое допустить, но для ученых — нет. Реальность состоит не только из чисел.

Полученный результат следует рассматривать как доказательство того, что системы счисления являются единицами измерения чисел. Ведь мы не можем сравнивать числа с разными единицами измерения. Если одни и те же действия с разными единицами измерения одной и той же величины приводят к разным результатам после их сравнения, значит это не имеет ничего общего с математикой.

Что же такое настоящая математика? Это когда результат математического действия не зависит от величины числа, применяемой единицы измерения и от того, кто это действие выполняет.

Открывает дверь и говорит:

Ой! А это разве не женский туалет?
— Девушка! Это лаборатория по изучению индефильной святости душ при вознесении на небеса! Нимб сверху и стрелочка вверх. Какой еще туалет?

Женский… Нимб сверху и стрелочка вниз — это мужской.

Если у вас перед глазами несколько раз в день мелькает вот такое вот произведение дизайнерского искусства,

Тогда не удивительно, что в своем автомобиле вы вдруг обнаруживаете странный значок:

Лично я делаю над собой усилие, чтобы в какающем человеке (одна картинка), увидеть минус четыре градуса (композиция из нескольких картинок: знак минус, цифра четыре, обозначение градусов). И я не считаю эту девушку дурой, не знающей физику. Просто у неё дугой стереотип восприятия графических образов. И математики нас этому постоянно учат. Вот пример.

1А — это не «минус четыре градуса» или «один а». Это «какающий человек» или число «двадцать шесть» в шестнадцатеричной системе счисления. Те люди, которые постоянно работают в этой системе счисления, автоматически воспринимают цифру и букву как один графический символ.

Надеюсь, вы уже прочитали про числовую окружность и знаете, почему она называется числовой, где на ней начало координат и в какой стороне положительное направление. Если нет, то бегом ! Если вы, конечно, собираетесь находить точки на числовой окружности.

Обозначаем числа \(2π\), \(π\), \(\frac{π}{2}\), \(-\frac{π}{2}\), \(\frac{3π}{2}\)

Как вы знаете из прошлой статьи, радиус числовой окружности равен \(1\). Значит, длина окружности равняется \(2π\) (вычислили по формуле \(l=2πR\)). С учетом этого отметим \(2π\) на числовой окружности. Чтобы отметить это число нужно пройти от \(0\) по числовой окружности расстояние равно \(2π\) в положительном направлении, а так как длина окружности \(2π\), то получается, что мы сделаем полный оборот. То есть, числу \(2π\) и \(0\) соответствует одна и та же точка. Не переживайте, несколько значений для одной точки — это нормально для числовой окружности.

Теперь обозначим на числовой окружности число \(π\). \(π\) – это половина от \(2π\). Таким образом, чтобы отметить это число и соответствующую ему точку, нужно пройти от \(0\) в положительном направлении половину окружности.


Отметим точку \(\frac{π}{2}\) . \(\frac{π}{2}\) – это половина от \(π\), следовательно чтобы отметить это число, нужно от \(0\) пройти в положительном направлении расстояние равное половине \(π\), то есть четверть окружности.


Обозначим на окружности точки \(-\)\(\frac{π}{2}\) . Двигаемся на такое же расстояние, как в прошлый раз, но в отрицательном направлении.


Нанесем \(-π\). Для этого пройдем расстояние равное половине окружности в отрицательном направлении.


Теперь рассмотрим пример посложнее. Отметим на окружности число \(\frac{3π}{2}\) . Для этого дробь \(\frac{3}{2}\) переведем в \(\frac{3}{2}\) \(=1\)\(\frac{1}{2}\) , т.е. \(\frac{3π}{2}\) \(=π+\)\(\frac{π}{2}\) . Значит, нужно от \(0\) в положительную сторону пройти расстояние в пол окружности и еще в четверть.


Задание 1 . Отметьте на числовой окружности точки \(-2π\),\(-\)\(\frac{3π}{2}\) .

Обозначаем числа \(\frac{π}{4}\), \(\frac{π}{3}\), \(\frac{π}{6}\) ,\(\frac{7π}{6}\), \(-\frac{4π}{3}\), \(\frac{7π}{4}\)

Выше мы нашли значения в точках пересечения числовой окружности с осями \(x\) и \(y\). Теперь определим положение промежуточных точек. Для начала нанесем точки \(\frac{π}{4}\) , \(\frac{π}{3}\) и \(\frac{π}{6}\) .
\(\frac{π}{4}\) – это половина от \(\frac{π}{2}\) (то есть, \(\frac{π}{4}\) \(=\)\(\frac{π}{2}\) \(:2)\) , поэтому расстояние \(\frac{π}{4}\) – это половина четверти окружности.


\(\frac{π}{4}\) – это треть от \(π\) (иначе говоря,\(\frac{π}{3}\) \(=π:3\)), поэтому расстояние \(\frac{π}{3}\) – это треть от полукруга.

\(\frac{π}{6}\) – это половина \(\frac{π}{3}\) (ведь \(\frac{π}{6}\) \(=\)\(\frac{π}{3}\) \(:2\)) поэтому расстояние \(\frac{π}{6}\) – это половина от расстояния \(\frac{π}{3}\) .


Вот так они расположены друг относительно друга:

Замечание: Расположение точек со значением \(0\), \(\frac{π}{2}\) ,\(π\), \(\frac{3π}{2}\) , \(\frac{π}{4}\) , \(\frac{π}{3}\) , \(\frac{π}{6}\) лучше просто запомнить. Без них числовая окружность, как компьютер без монитора, вроде бы и полезная штука, а использовать крайне неудобно.

Давайте теперь обозначим на окружности точку \(\frac{7π}{6}\) , для этого выполним следующие преобразования: \(\frac{7π}{6}\) \(=\)\(\frac{6π + π}{6}\) \(=\)\(\frac{6π}{6}\) \(+\)\(\frac{π}{6}\) \(=π+\)\(\frac{π}{6}\) . Отсюда видно, что от нуля в положительную сторону надо пройти расстояние \(π\), а потом еще \(\frac{π}{6}\) .


Отметим на окружности точку \(-\)\(\frac{4π}{3}\) . Преобразовываем: \(-\)\(\frac{4π}{3}\) \(=-\)\(\frac{3π}{3}\) \(-\)\(\frac{π}{3}\) \(=-π-\)\(\frac{π}{3}\) . Значит надо от \(0\) пройти в отрицательную сторону расстояние \(π\) и еще \(\frac{π}{3}\) .


Нанесем точку \(\frac{7π}{4}\) , для этого преобразуем \(\frac{7π}{4}\) \(=\)\(\frac{8π-π}{4}\) \(=\)\(\frac{8π}{4}\) \(-\)\(\frac{π}{4}\) \(=2π-\)\(\frac{π}{4}\) . Значит, чтобы поставить точку со значением \(\frac{7π}{4}\) , надо от точки со значением \(2π\) пройти в отрицательную сторону расстояние \(\frac{π}{4}\) .


Задание 2 . Отметьте на числовой окружности точки \(-\)\(\frac{π}{6}\) ,\(-\)\(\frac{π}{4}\) ,\(-\)\(\frac{π}{3}\) ,\(\frac{5π}{4}\) ,\(-\)\(\frac{7π}{6}\) ,\(\frac{11π}{6}\) , \(\frac{2π}{3}\) ,\(-\)\(\frac{3π}{4}\) .

Обозначаем числа \(10π\), \(-3π\), \(\frac{7π}{2}\) ,\(\frac{16π}{3}\), \(-\frac{21π}{2}\), \(-\frac{29π}{6}\)

Запишем \(10π\) в виде \(5 \cdot 2π\). Вспоминаем, что \(2π\) – это расстояние равное длине окружности, поэтому чтобы отметить точку \(10π\), нужно от нуля пройти расстояние равное \(5\) окружностям. Нетрудно догадаться, что мы окажемся снова в точке \(0\), просто сделаем пять оборотов.


Из этого примера можно сделать вывод:

Числам с разницей в \(2πn\), где \(n∈Z\) (то есть \(n\) — любое целое число) соответствует одна и та же точка.

То есть, чтобы поставить число со значением больше \(2π\) (или меньше \(-2π\)), надо выделить из него целое четное количество \(π\) (\(2π\), \(8π\), \(-10π\)…) и отбросить. Тем самым мы уберем из числа, не влияющие на положение точки «пустые обороты».

Еще один вывод:

Точке, которой соответствует \(0\), также соответствуют все четные количества \(π\) (\(±2π\),\(±4π\),\(±6π\)…).

Теперь нанесем на окружность \(-3π\). \(-3π=-π-2π\), значит \(-3π\) и \(–π\) находятся в одном месте на окружности (так как отличаются на «пустой оборот» в \(-2π\)).


Кстати, там же будут находиться все нечетные \(π\).

Точке, которой соответствует \(π\), также соответствуют все нечетные количества \(π\) (\(±π\),\(±3π\),\(±5π\)…).

Сейчас обозначим число \(\frac{7π}{2}\) . Как обычно, преобразовываем: \(\frac{7π}{2}\) \(=\)\(\frac{6π}{2}\) \(+\)\(\frac{π}{2}\) \(=3π+\)\(\frac{π}{2}\) \(=2π+π+\)\(\frac{π}{2}\) . Два пи – отбрасываем, и получается что, для обозначения числа \(\frac{7π}{2}\) нужно от нуля в положительную сторону пройти расстояние равное \(π+\)\(\frac{π}{2}\) (т.е. половину окружности и еще четверть).

(10-й класс)

Цель. Показать учащимся приём построения “табличных” и связанных с ними углов без транспортира. Научить записывать значения углов, соответствующих указанным точкам единичной окружности.

Оборудование.

  1. Модель единичной окружности (плакат).
  2. Плакат единичной окружности, где показаны приёмы построения “табличных” углов.
  3. Карточки самостоятельных работ.
  4. Карточки с домашними заданиями.
  5. Карточки – “считалочки”.
  6. Геометрические инструменты.
  7. Фломастеры, цветной мел.
  8. Кодоскоп.

I. Организационный момент.

Постановка цели, мотивация учения.

Чтобы лучше понять и запомнить расположение точек на единичной окружности, мы познакомимся с приёмами построения “табличных” (30°, 45°, 60°) и связанных с ними углов без транспортира. Это позволит в дальнейшем не только легче освоить радианную меру угла, но и быстрее находить значения тригонометрических функций, хорошо решать простейшие тригонометрические уравнения, неравенства, системы.

II. Новый материал.

(фронтальная форма учебной работы)

1.1. Начертите на определённых листах и на доске координатную плоскость и окружность с центром в начале координат радиусом равным 1.

1. 2. Определение единичной окружности (учащиеся)

1.3. Понятие узловых точек (пересечения единичной окружности и осей координат)

2.1. Отметим угловые точки на единичной окружности и запишем соответствующие им углы (0°, 90°, 180°, 360°)

(учащиеся работают у доски и на своих моделях единичной окружности).

Положительные углы против хода часовой стрелки (одним цветом).

Отрицательные углы – по часовой стрелке (другим цветом).

Все углы записываем внутри окружности.

3.1. Как построить точки, соответствующие углам 45°, 135°, 225°, 315°?

(делением пополам координатных углов).

3.2. Учащиеся предлагают свои варианты. Затем на отдельно приготовленном плакате рассказывают приём построение точек, соответствующие углам 45°, 135°, 225°, 315°.

3.3. Данный приём применяется к единичной окружности на доске и к своим моделям. Отмечают точки, соответствующие углам 45°, 135°, 225°, 315°.

4.1. Как построить точки соответствующие углам 30°, 150°, 210°, 330°?

(делением пополам вертикальных радиусов).

4.2. Учащиеся предлагают свои варианты. Затем по готовому плакату объясняют построение данных углов.

4.3. На демонстрационной модели и своих моделях единичных окружностей отмечают точки, соответствующие углам 30°, 150°, 210°, 330°.

5.1. Как построить точки соответствующие углам 60°, 120°, 240°, 300°?

(делением пополам горизонтальных радиусов).

5.2. Учащиеся предлагают свои варианты. Затем по готовому плакату объясняют приём построения данных углов

5.3. Учащиеся отмечают данные углы на демонстрационной модели и на своих моделях, используя предложенный приём.

6.1. Выразим в радианной мере величины углов

6.2. Около каждой из отмеченных точек единичной окружности запишем им соответствующие углы в радианах. (Вычисления на доске. Пример.)

(неотрицательные числа пишем одним цветом, а отрицательные другим).

7.1. Запоминанию данных углов помогает “Считалка”.

(карточки со “ Считалками” разложены на ученических столах перед началом урока).

а) “Ра пи на два” (/2)

“Два пи на два” ()

“Три пи на два” (3/2)

б) “Раз пи на четыре” (/4)

“Два пи на четыре” (2/4)

“Три пи на четыре” (3/4)

8. Запись углов, соответствующих одной точке единичной окружности

Пусть на окружности дана точка Р, которая получается повтором точки Р 0 на угол .

При обходе окружности на целое число оборотов мы попадаем на исходную точку Р. Значит, точке Р наравне с числом соответствует любое число вида +2п , п ЄZ.

На единичной окружности отмечены точки, соответствующие углам, запишите все такие углы, используя градусную меру и радианную.

0 =45 0 , любой другой угол отличается от угла 0 на 360 0 п , п ЄZ.

Запишем: =45 0 +360 0 п, п Є Z;

III. Проверка усвоения изученного.

(самостоятельная работа)

Для всех учащихся карточки с заданиями самостоятельной работы (записываем только ответы).

Самостоятельная работа.

1.На единичной окружности отмечены точки, соответствующие углам и , заключённым в промежутке от 0 0 до 360 0 . Выразите углы и в градусах.

2. На единичной окружности отмечены точки, соответствующие углам и , заключённым в промежутке от 0 до 2 радиан. Выразите углы и , в радианах

3.На единичной окружности отмечены точки, соответствующие углам и , заключённым в промежутке от 0 до 2 радиан. Выразите и в радианах.

4. На единичной окружности отмечены точки, соответствующие углам и . Запишите все углы и , используя градусную меру.

Урок и презентация на тему: «Числовая окружность на координатной плоскости»

Дополнительные материалы
Уважаемые пользователи, не забывайте оставлять свои комментарии, отзывы, пожелания! Все материалы проверены антивирусной программой.

Пособия и тренажеры в интернет-магазине «Интеграл» для 10 класса от 1С
Алгебраические задачи с параметрами, 9–11 классы
Решаем задачи по геометрии. 2 = 1, \\ x = y. \end {cases}$
Решив данную систему, получаем: $y = x =\frac{\sqrt{2}}{2}$.
Значит, координаты точки M, соответствующей числу $\frac{π}{4}$, будут $M(\frac{π}{4})=M(\frac{\sqrt{2}}{2};\frac{\sqrt{2}}{2})$.
Аналогичным образом рассчитываются координаты точек, представленных на предыдущем рисунке.

Координаты точек числовой окружности



Рассмотрим примеры

Пример 1.
Найти координату точки числовой окружности: $Р(45\frac{π}{4})$.

Решение:
$45\frac{π}{4} = (10 + \frac{5}{4}) * π = 10π +5\frac{π}{4} = 5\frac{π}{4} + 2π*5$.
Значит, числу $45\frac{π}{4}$ соответствует та же точка числовой окружности, что и числу $\frac{5π}{4}$. Посмотрев значение точки $\frac{5π}{4}$ в таблице, получаем: $P(\frac{45π}{4})=P(-\frac{\sqrt{2}}{2};-\frac{\sqrt{2}}{2})$.

Пример 2.
Найти координату точки числовой окружности: $Р(-\frac{37π}{3})$.

Решение:

Т.к. числам $t$ и $t+2π*k$, где k-целое число, соответствует одна и та же точка числовой окружности то:
$-\frac{37π}{3} = -(12 + \frac{1}{3})*π = -12π –\frac{π}{3} = -\frac{π}{3} + 2π*(-6)$.
Значит, числу $-\frac{37π}{3}$ соответствует та же точка числовой окружности, что и числу $–\frac{π}{3}$, а числу –$\frac{π}{3}$ соответствует та же точка, что и $\frac{5π}{3}$. Посмотрев значение точки $\frac{5π}{3}$ в таблице, получаем:
$P(-\frac{37π}{3})=P(\frac{{1}}{2};-\frac{\sqrt{3}}{2})$.

Пример 3.
Найти на числовой окружности точки с ординатой $у =\frac{1}{2}$ и записать, каким числам $t$ они соответствуют?

Решение:
Прямая $у =\frac{1}{2}$ пересекает числовую окружность в точках М и Р. Точка М соответствует числу $\frac{π}{6}$ (из данных таблицы). Значит, и любому числу вида: $\frac{π}{6}+2π*k$. Точка Р соответствует числу $\frac{5π}{6}$, а значит, и любому числу вида $\frac{5π}{6} +2 π*k$.
Получили, как часто говорят в таких случаях, две серии значений:
$\frac{π}{6} +2 π*k$ и $\frac{5π}{6} +2π*k$.
Ответ: $t=\frac{π}{6} +2 π*k$ и $t=\frac{5π}{6} +2π*k$.

Пример 4.
Найти на числовой окружности точки с абсциссой $x≥-\frac{\sqrt{2}}{2}$ и записать, каким числам $t$ они соответствуют.

Решение:

Прямая $x =-\frac{\sqrt{2}}{2}$ пересекает числовую окружность в точках М и Р. Неравенству $x≥-\frac{\sqrt{2}}{2}$ соответствуют точки дуги РМ. Точка М соответствует числу $3\frac{π}{4}$ (из данных таблицы). Значит, и любому числу вида $-\frac{3π}{4} +2π*k$. Точка Р соответствует числу $-\frac{3π}{4}$, а значит, и любому числу вида $-\frac{3π}{4} +2π*k$.

Тогда получим $-\frac{3π}{4} +2 π*k ≤t≤\frac{3π}{4} +2πk$.

Ответ: $-\frac{3π}{4} +2 π*k ≤t≤\frac{3π}{4} +2πk$.

Задачи для самостоятельного решения

1) Найти координату точки числовой окружности: $Р(\frac{61π}{6})$.
2) Найти координату точки числовой окружности: $Р(-\frac{52π}{3})$.
3) Найти на числовой окружности точки с ординатой $у = -\frac{1}{2}$ и записать, каким числам $t$ они соответствуют.
4) Найти на числовой окружности точки с ординатой $у ≥ -\frac{1}{2}$ и записать, каким числам $t$ они соответствуют.
5) Найти на числовой окружности точки с абсциссой $x≥-\frac{\sqrt{3}}{2}$ и записать, каким числам $t$ они соответствуют.

Табличка на двери

Внеклассный урок — Тригонометрические свойства чисел числовой окружности

Тригонометрические свойства чисел числовой окружности

Свойство 1:

 

sin (–t) = –sin t

 

 

cos (–t) = cos t

 

tg (–t) = –tg t

 

ctg (–t) = –ctg t

Пояснение. Пусть t = –60º  и  t = –210º (см.рисунок).

cos –60º равен 1/2. Но cos 60º тоже равен 1/2. То есть косинусы –60º  и 60º  равны как по модулю, так и по знаку: cos –60º = cos 60º.

cos –210º равен –√3/2. Но cos 210º тоже равен –√3/2. То есть: cos –210º = cos 210º.

Таким образом, мы доказали, что cos (–t) = cos t.

sin –60º равен –√3/2. А sin 60º равен √3/2. То есть sin –60º и sin 60º равны по модулю, но противоположны по знаку.

sin –210º равен 1/2. А sin 210º равен –1/2. То есть sin –210º и sin 210º равны по модулю, но противоположны по знаку.

Таким образом, мы доказали, что sin (–t) = –sin t.

 Посмотрите, что происходит с тангенсами и котангенсами этих углов – и вы сами легко докажете себе верность двух других тождеств, приведенных в таблице.

Вывод: косинус – четная функция, синус, тангенс и котангенс – нечетные функции.

 

Свойство 2: Так как t = t + 2πk, то:

 

sin (t + 2πk) = sin t

 

 

cos (t + 2πk) = cos t

Пояснение: t и t + 2πk – это одна и та же точка на числовой окружности. Просто в случае с 2πk  мы совершаем определенное количество полных оборотов по окружности, прежде чем приходим к точке t. Значит, и равенства, изложенные в этой таблице, очевидны.

 

Свойство 3: Если две точки окружности находятся друг против друга относительно центра О, то их синусы и косинусы равны по модулю, но противоположны по знаку, а их тангенсы и котангенсы одинаковы как по модулю, так и по знаку.

 

sin (t + π) = –sin t

 

 

cos (t + π) = –cos t

 

tg (t + π) = tg t

 

ctg (t + π) = ctg t

Пояснение: Пусть точка М находится в первой четверти (см. рисунок ниже). Она имеет положительное значение синуса и косинуса. Проведем от этой точки диаметр – то есть отрезок, проходящий через центр оси координат и заканчивающийся  в точке окружности напротив. Обозначим эту точку буквой N. Как видите, дуга MN равна половине окружности. Мы знаем, что половина окружности – это величина, равная π. Значит, точка N находится на расстоянии π от точки М. Говоря иначе, если к точке М прибавить расстояние π, то мы получим точку N, находящуюся напротив:

М + π = N.

Точка N имеет те же значения синуса и косинуса, что и точка М, но с противоположным знаком (минус). То есть синус точки N равен синусу точки М с противоположным знаком, косинус точки N также равен косинусу точки М, но тоже с противоположным знаком:

sin (M + π) = –sin M, cos (M + π) = –cos M.

Или обобщая:

sin (t + π) = –sin t, cos (t + π) = –cos t.

Мы доказали, что синус и косинус диаметрально противоположных точек равны по модулю, но противоположны по знаку.

 

 Идем дальше. Тангенс и котангенс точки М имеют положительное значение. Но вычислите тангенс и котангенс точки N (разделите ее синус на косинус и косинус на синус) и увидите, что они тоже со знаком плюс.

Значит:

                     –sin t
tg (t + π) = ———— = tg t
                    –cos t

 

                      –cos t
ctg (t + π) = ———— = ctg t
                      –sin t

Мы доказали, что тангенс и котангенс диаметрально противоположных точек окружности имеют не только одинаковое значение, но и одинаковый знак.

 

Свойство 4: Если две точки окружности находятся в соседних четвертях, а расстояние между точками равно одной четверти окружности, то синус одной точки равен косинусу другой с тем же знаком, а косинус одной точки равен синусу второй с противоположным знаком.

    

тригонометрия — Понимание синуса, косинуса и тангенса в единичной окружности

спросил

Изменено 2 года, 6 месяцев назад

Просмотрено 3к раз

$\begingroup$

На следующей диаграмме показано, как использовать угол $\theta$ для нахождения косинуса и синуса. Однако мне трудно представить, как прийти к касательной. Кроме того, правда ли, что во всех тригонометрических соотношениях прямоугольных треугольников нам всегда нужно использовать один из непрямых углов?

  • тригонометрия

$\endgroup$

3

$\begingroup$

Если вы обратите внимание, меньший левый прямоугольный треугольник подобен большему прямоугольному треугольнику, примыкающему к нему, поскольку они оба являются прямоугольными треугольниками с углами измерения $\frac{\pi}{2}$, $\theta$ и $(\frac{\pi}{2}-\theta)$.

Если два треугольника подобны, отношение их соответствующих сторон будет равным.

$$\frac{\cos\theta}{1}$$ В меньшем прямоугольном треугольнике $\cos\theta$ лежит напротив угла $(\frac{\pi}{2}-\theta)$, а $1$ является гипотенузой.

В большом прямоугольном треугольнике $\sin\theta$ противоположно углу $(\frac{\pi}{2}-\theta)$, а $x$ (неизвестная переменная) является гипотенузой.

$$\frac{\sin\theta}{x}$$

Используя правила подобия, мы можем сказать, что эти два отношения эквивалентны.

$$\frac{\cos\theta}{1} = \frac{\sin\theta}{x}$$ $$x\cos\theta = \sin\theta\имеется в виду x = \frac{\sin\theta}{\cos\theta} =\имеется в виду \boxed{x = \tan\theta}$$

На этой странице вы можете найти $\csc\theta$, $\sec\theta$ и $\cot\theta$: графическое представление триггерных функций.

$\endgroup$

2

$\begingroup$

  • В первом и третьем квадрантах $\tan(\theta)$ — это длина от $(\cos(\theta),\sin(\theta))$ до оси $x$ вдоль линии, касательной к круг в $(\cos(\theta),\sin(\theta))$. Во втором и четвертом квадрантах ситуация практически такая же, но мы используем противоположный знак.
  • Можно представить $\cos(\pi/2)$ и $\sin(\pi/2)$ с точки зрения треугольника как отношения вырожденного треугольника с углами $(\pi/2,\pi /2,0)$, что на самом деле является просто отрезком. Теперь вы идентифицируете один из $\pi/2$ как «прямой» угол, а другой как «острый» угол и измеряете отношения относительно этого «острого» угла.

$\endgroup$

$\begingroup$

Угол между красной и синей линиями равен $\theta$. Итак, в этом треугольнике $$\cos\theta=\frac{\rm red}{\rm blue}$$ Or $${\rm blue}=\frac{\rm red}{\cos\theta}=\ frac{\sin\theta}{\cos\theta}=\tan\theta$$

$\endgroup$

2

$\begingroup$

Запомните эту таблицу SohCahToa , чтобы вы всегда могли ее записать:

Для $sin$ и $cos$ в единичном круге вы можете поместить значения в таблицу над большим $1 $ для гипотенузы.

Пример: подумайте о

$sin(\frac{\pi}{6})= \frac{{\frac{\sqrt1}{2}}}{1}$

$cos(\frac{\pi}{6})= \frac{{\frac{\sqrt3}{2}}}{1}$

и размещение единичной окружности $\left(cos( \frac{\pi}{6}),sin(\frac{\pi}{6})\right)$.

$\endgroup$

Твой ответ

Зарегистрируйтесь или войдите

Зарегистрироваться через Google

Зарегистрироваться через Facebook

Зарегистрируйтесь, используя электронную почту и пароль

Опубликовать как гость

Электронная почта

Требуется, но никогда не отображается

Опубликовать как гость

Электронная почта

Требуется, но не отображается

Нажимая «Опубликовать свой ответ», вы соглашаетесь с нашими условиями обслуживания, политикой конфиденциальности и политикой использования файлов cookie

.

Окружность радиусом 1.

Произношение: /ˈju.nɪt ˈsɜr.kəl/ Объяснить

круг с радиус ровно 1 . [1] Единичный круг используется для определения функций синуса и косинуса.

Рисунок 1: Единичный круг

Связь между единичным кругом и тригонометрическими функциями

В тригонометрии прямоугольного треугольника синус определяется как противоположность / гипотенуза, и косинус определяется как смежная/гипотенуза. Глядя на единичный круг, радиус равен гипотенуза прямоугольного треугольника. Красная сторона прямоугольного треугольника (см. манипуляцию 1) это соседняя нога. Таким образом, косинус равен прилежащему/гипотенузе. Но в единичный круг, гипотенуза всегда равна 1. Таким образом, косинус всегда равен длина прилежащего катета прямоугольного треугольника. По аналогичному аргументу синус всегда равен длине синего катета, противоположного катету.

Круговые тригонометрические тождества

Нажмите на синие точки и перетащите их, чтобы изменить фигуру.

Манипулятивное 2 — Тригонометрия круга Создано с помощью GeoGebra.

Тригонометрические тождества окружности – это уравнения, которые можно использовать для определения тригонометрических функций. Дана окружность с центром в начале координат, тригонометрические функции определяются с помощью координаты x точки на круг ( x ), координата y точки на окружности ( y ), радиус окружности ( r ) и угол поворота от пересечения окружности и оси x ( θ ).

. 0088
  1. МакАдамс, Дэвид Э.. Словарь всех математических слов, единичный круг . Издание 2-го класса 20150108-4799968. стр. 186. Life is a Story Problem LLC. 8 января 2015. Купить книгу
  2. единичный круг . www.merriam-webster.com. Британская энциклопедия. Мерриам-Вебстер. Последний доступ 17.12.2018. http://www.merriam-webster.com/dictionary/unit Circle. Купить книгу

Дополнительная информация

  • МакАдамс, Дэвид Э.. Круг . allmathwords.org. Вся энциклопедия математических слов. ООО «Жизнь — это проблема истории». 12.03.2009. https://www.allmathwords.org/en/c/circle.html.

Цитируйте эту статью как:

МакАдамс, Дэвид Э. Unit Circle . 21.12.2018. Вся энциклопедия математических слов. ООО «Жизнь — это проблема истории». https://www.allmathwords.org/en/u/unitcircle.html.

Кредиты изображений

  • Все изображения и манипуляции принадлежат Дэвиду МакАдамсу, если не указано иное. Все изображения Дэвида МакАдамса защищены авторским правом © Life is a Story Problem LLC и находятся под лицензией Creative Commons Attribution-ShareAlike 4.0 International License.

История изменений

21.12.2018:

Пересмотрено и исправлено произношение IPA.

(МакАдамс, Дэвид Э.)

17.12.2018:

Удалены битые ссылки, обновлена ​​лицензия, реализована новая разметка, реализованы новые приложения GeoGebra.

(МакАдамс, Дэвид Э.)

07.08.2018:

Словарные ссылки изменены на формат WORDLINK.

(МакАдамс, Дэвид Э.)

09.01.2010:

Добавлен раздел «Ссылки».

(МакАдамс, Дэвид Э.)

27.11.2008:

Добавлены круговые тригонометрические тождества.

(МакАдамс, Дэвид Э.)

15.09.2008:

Расширенный раздел «Дополнительная информация».

(МакАдамс, Дэвид Э.)

15.06.2008:

Добавлена ​​манипулятивная связь между тригонометрическими функциями и единичным кругом. (

(МакАдамс, Дэвид Э.)

11.03.2008:

Изменено См. также для соответствия текущим стандартам.

(МакАдамс, Дэвид Э.)

18. 08.2007:

Начальная версия.

(МакАдамс, Дэвид Э.)

Единичный круг — Разделы по тригонометрии

Темы | Дом

 

15

Определения

Знаки в каждом квадранте

Квадрантальные углы

Единица окружности

Радианная мера

АНАЛИТИЧЕСКАЯ ТРИГОНОМЕТРИЯ является расширением тригонометрии прямоугольного треугольника. Это происходит на плоскости x y . Поскольку тригонометрия фактически используется в вычислениях и физике, речь не идет о решении треугольников. Оно становится математическим описанием вещей, которые вращаются или вибрируют, таких как свет, звук, траектории планет вокруг солнца или спутников вокруг земли. Поэтому необходимо иметь углы любой величины и распространять на них значения тригонометрических функций. Мы делаем это сейчас.

Пусть радиус длиной r образует угол θ в стандартном положении, и пусть его конечная точка имеет координаты ( x , ​​ y ). Возникает вопрос: как мы теперь определим шесть тригонометрических функций θ?

Мы возьмем пример с первого квадранта. В этом квадранте

радиус r закончится в точке ( x , ​​ y ). Эти координаты определяют прямоугольный треугольник. Далее следуют прямоугольные определения шести тригонометрических функций.

Тригонометрические тождества окружностей
sin θ  =   у
р
        csc θ  =   р
у
 
cos θ  =   х
р
        сек θ  =   р
х
 
тан θ  =   г
х
        детская кроватка θ  =   х
у

Согласно теореме Пифагора,

Таким образом, мы распространяем смысл тригонометрических функций на углы, которые оканчиваются в любом квадранте . Это с точки зрения координат ( x , ​​ y ) конечной точки на расстоянии r от начала координат.

Но прежде чем мы приведем пример, подумайте над этим вопросом:

Будет ли функция θ зависеть от длины r ?

Чтобы увидеть ответ, наведите указатель мыши на цветную область.
Чтобы снова закрыть ответ, нажмите «Обновить» («Reload»).
Сначала ответь на вопрос сам!

Нет, не будет. Функции определяются как отношения сторон, а не их длины.

Предположим, что AB и AC — два разных радиуса. Но треугольники ABD, ACE подобны. Пропорционально,

ДБ : ВА = EC : CA

sin θ — напротив гипотенузы — не зависит от длины радиуса. И аналогично для остальных функций. Поэтому мы можем выбрать любой радиус, какой пожелаем. Обычно мы берем r = 1. Это называется единичным кругом.

Тригонометрические функции на самом деле зависят только от угла θ — и именно поэтому мы говорим, что они являются функциями θ.

Пример 1.   Прямая, вставленная в начало координат, заканчивается в точке (3, 2), поскольку она образует угол θ в стандартном положении. Оцените все шесть функций θ.

Ответить . x = 3, y = 2. Следовательно, согласно определениям:

sin θ  =  у
р
 =  2
  csc θ  =  р
у
 = 
  2
 
cos θ  =  x
р
 =  3
  сек θ  =  р
х
 = 
  3
 
тан θ  =  г
х
 =  2
3
  детская кроватка θ  =  х
у
 =  3
2

Задача 1. Прямая из начала координат заметает угол θ и заканчивается в точке (3, −4). Оцените шесть функций θ.

х = 3, у = -4. Следовательно,

sin θ  = −  4
5
        csc θ  = −  5
4
 
cos θ  =  3
5
        сек θ  =  5
3
 
загар θ  = −  4
3
        детская кроватка θ  = −  3
4

Задача 2. Знаки в каждом квадранте.

а) Алгебраический знак sin θ всегда будет знаком какой
а) координаты? г , потому что sin θ = y / r , а r всегда положительно.

а) Следовательно, в каких квадрантах sin θ — y — будет положительным? I и II.

а)  В каких квадрантах sin θ будет отрицательным? III и IV.

b) Алгебраический знак cos θ всегда будет знаком какой
b)  координаты? x , ​​потому что θ = x / r , и снова r всегда

положительный.

а) Следовательно, в каких квадрантах cos θ — x — будет положительным? I и IV.

а)  В каких квадрантах cos θ будет отрицательным? II и III.

c) В каких квадрантах алгебраический знак тангенса θ ( y / x ) будет положительным?

I и III. x и y будут иметь одинаковые знаки.

d)  В каких квадрантах алгебраический знак тангенса θ будет отрицательным?

II и IV. x и y будут иметь противоположные знаки.

д) csc θ будет иметь тот же знак, что и какая другая функция?

sin θ, потому что они обратны.

е) сек θ будет иметь тот же знак, что и какая другая функция?

cos θ

g) cot θ будет иметь тот же знак, что и какая другая функция?

желтовато-коричневый θ

Квадрантные углы

Квадрантальный угол — это угол, который заканчивается на осью x или y .

Проблема 3.

а)  Каковы квадрантные углы в градусах?

0°, 90°, 180°, 270°; и котерминальные с ними углы.

b)  Каковы квадрантные углы в радианах?

0,
2
, π,
 2
; и котерминальные с ними углы.

c) Когда угол заканчивается на оси x , ​​каково значение
c)   y -координата? 0.  На x — ось, y  = 0,

d) Когда угол заканчивается на оси y , ​​каково значение
c)   x -координата? 0.   На оси y x  = 0,

Теперь арифметическим фактом является то, что не существует числа со знаменателем 0.

нет
0
  не имеет значения.

Следовательно, везде, где знаменатель тригонометрической функции — x  или y — равен 0, при таком квадрантном угле функция не будет иметь значения.

Например,

желтовато-коричневый θ = г
х

Везде, где x = 0, тангенс θ не будет иметь значения. Где х = 0? Когда угол заканчивается на и -ось.

tan θ не имеет значения при θ = и при , или при −, что котерминально.

Эти значения θ будут особенностями тангенса θ. (Тема 18 Предварительного исчисления.)

В математическом анализе учащийся встретится с выражением: «Предел x приближается к 0 = ∞».

Как объяснялось в Теме 4 исчисления, слово бесконечность вместе с его символом ∞ не является числом и не местом. Когда мы говорим, что функция становится бесконечной, мы просто имеем в виду, что нет предела ее значениям. Функция будет иметь значения больше, чем любое число, которое мы назовем.

Обратите внимание, что мы говорим: «Поскольку x приближается к 0». То есть x могут иметь значения, близкие к 0, насколько нам угодно. Но x не может равняться 0.

Задача 4.   Для каких квадрантных углов следующие функции не имеют значения?

а) детская кроватка θ    детская кроватка θ = х
г
. Поэтому всякий раз, когда y = 0, то есть на

ось x — кроватка θ не будет иметь значения. cot θ не будет иметь значения при θ = 0 и θ = π.

б)  сек θ  

сек θ = р
х
. Следовательно, sec θ не будет иметь значения там, где x = 0,
, который находится на оси y . сек θ не будет иметь значения при θ = 90 126
2
и θ = 3 π
 2
.

в)  sin θ  

sin θ = у
р
. Но r никогда не равно 0. Нет угла, для которого

sin θ не имеет значения.

Единица окружности

Тригонометрические функции являются функциями только угла θ. Поэтому мы можем выбрать любой радиус, какой пожелаем, и самым простым является круг радиуса 1, единичный круг.

На единичном круге функции имеют особенно простую форму. Например,

sin θ  =   г
1
  =  y .
потому что θ  =   х
1
 =  x .

Значение sin θ равно y-координате конечной точки единичного радиуса.

Значение cos θ является координатой x .

Ниже показаны координаты конечной точки единичной окружности каждого квадрантного угла:

Если функция имеет значение в квадрантном угле,
это может быть только 0, 1 или −1.

Рассмотрим sin θ для каждого квадрантного угла. Мы только что видели, что значение sin θ равно y -координата:

sin θ = y .

Следовательно, для каждого квадрантного угла значение sin θ — y — равно 0, 1 или −1.

При θ = 0,   sin θ  =  0,
 
При θ =  
2
, sin θ = 1,
 
При θ = π,   sin θ  =  0,
 
При θ  = 
 2
, sin θ = −1.

Чтобы оценить функцию в квадрантном угле, учащийся должен начертить единичный круг.

Проблема 5.   Оцените следующее. Нет столов.

а)  cos 0°

cos 0° = 1.  cos θ равно x -координата.

б)   cos 90° = 0 в) cos 180° = −1

г)   cos 270° = 0 

д) тангенс 0°

тангенс 0° = 0. тангенс θ равен y / x = 0/1 = 0,

е) тангенс 90° 1/0 не имеет значения.

ж) тангенс 180° = 0 

ч) tan 270° не имеет значения.

Проблема 6.   Оцените следующее — если оно имеет значение. Нет столов.

  а)  кос 
2
б) грех
2
   = 1       в) грех    = 0       г)  cos     = −1 
д) детская кроватка 0    = x / y . Неважно.    е) детская кроватка
2
  = 0    
г) загар
2
  Нет значения. з) сек 0   = 1/ х = 1
i) csc (−
2
 = −1 к)  sin 2π  90 126 = 0
л) sin 3π = 0 л)  sin 4π  = 0
м)  sin (−π)  = 0 п) cos 2π = 1
о) cos 3π = -1 р) cos 4π = 1
   q)  cos 5π  = -1

Задача 7.    Объясните, почему мы можем написать следующее, где n может быть любым целым числом:

cos n π = (−1) n

(−1) n = ±1, в соответствии с тем, что n четно или нечетно. Если n четно (или 0), то cos n π сотерминально 0 радианам, и (−1) n  = 1. котерминальна π радианам, и (−1) n = -1.

См. единичный круг.

Радиан

Так как в любой окружности одно и то же отношение дуги к радиусу определяет единственный центральный угол, то для теоретической работы мы часто используем единичную окружность.

Радиан – это отношение длины дуги s к радиусу r . Следовательно, в единичном круге длина дуги s   составляет радиан. Длина этой дуги — действительное число 9.0306 х .

s = r θ = 1 ·   x = x .

Тогда мы можем определить меру в радианах как длину x дуги единичного круга. Сама окружность — 2π r — имеет радианную меру 2π.

Именно здесь термин тригонометрическая «функция» имеет свое полное значение. Для соответствия каждому вещественному числу x — каждой мере в радианах, каждой дуге — существует уникальное значение sin x , ​​cos x и так далее. Определение функции выполнено.

Кроме того, когда мы рисуем график y  = sin x , мы можем представить единичный круг, развернутый в обоих направлениях на ось x и таким образом обозначающий координаты π, 2π, −π , −2π и так далее по оси x .

Поскольку мера радиана может быть идентифицирована как дуга, обратные тригонометрические функции имеют свои имена. «arcsin» — это дуга — мера в радианах, синус которой равен определенному числу.

Отношение sin x
   x

Одна из основных математических теорем касается отношения

sin x
    x

для очень маленьких значений x .

В единичной окружности сторона AB противоположна углу AOB   это грех х .

грех х = АБ
 1
= АВ.

Мы можем видеть, что когда точка А на окружности очень близка к С, то есть когда центральный угол АОС чрезвычайно мал, тогда сторона АВ будет практически неотличима от длины дуги АС, т.е. мера радиана. То есть

грех x х
 
sin x
    x
1.

Следующая тема: Тригонометрические функции любого угла

Темы | Дом


Пожалуйста, сделайте пожертвование, чтобы TheMathPage оставался онлайн.
Даже 1 доллар поможет.


Copyright © 2022 Лоуренс Спектор

Вопросы или комментарии?

Электронная почта: [email protected]


Единичная окружность

Единичная окружность — это окружность радиусом 1 с центром в начале прямоугольной системы координат. Он обычно используется в контексте тригонометрии.

Когда луч проводится из начала координат единичной окружности, он пересекает единичную окружность в точке (x, y) и образует прямоугольный треугольник с осью x, как показано выше. Гипотенуза прямоугольного треугольника равна радиусу единичной окружности, поэтому она всегда будет равна 1. Следовательно, согласно теореме Пифагора, уравнение единичной окружности:

x 2 + y 2 = 1

Это верно для всех точек на единичной окружности, а не только в первом квадранте, и полезно для определения тригонометрических функций в терминах единичной окружности.

Определения единичной окружности тригонометрических функций

Единичная окружность часто используется в определении тригонометрических функций. Ниже приведен рисунок, показывающий все тригонометрические отношения, связанные с единичным кругом.

На рисунке выше точка A имеет координаты (x, y). Вместе с θ, углом, образованным между начальной стороной угла вдоль положительной оси x и конечной стороной угла, образованного поворотом луча против часовой стрелки, мы можем образовать прямоугольный треугольник. Используя тот факт, что радиус единичного круга равен 1 (и, следовательно, гипотенуза прямоугольного треугольника равна 1), мы можем использовать определения прямоугольного треугольника тригонометрических функций, чтобы найти, что , и . Исходя из этого, мы можем определить определения остальных тригонометрических функций, как показано в таблице ниже.

Trigonometric functions

Commonly used angles

While we can find trigonometric values ​​for any angle, some angles are worth remembering because of как часто они используются в тригонометрии. Углы 30°, 45° и 60°. В радианах они соответствуют соответственно. Ниже приведена таблица значений этих углов, а также рисунок значений на единичной окружности.

Angle sin(θ) cos(θ) tan(θ)
1

Как видно из таблицы или круга единиц выше, необходимо запомнить три значения: . Из-за природы единичного круга эти значения одинаковы для соответствующих углов в разных квадрантах единичного круга, с той лишь разницей, что их знаки основаны на квадранте, в котором находится угол. Следовательно, запоминание этих трех значений и того, как они соответствуют кратным 30°, 45° и 60°, что позволит вам заполнить все значения на единичном круге.

Другие углы на единичной окружности, которые следует запомнить, это те, конечные стороны которых лежат на оси x или y: 0° или 0 (что имеет эквивалентные значения синуса и косинуса как 360° или 2π), 90° или , 180 ° или π и, 270° или . При любом из этих углов sin(θ) или cos(θ) имеет значение –1, 0 или 1.

0 1 0
1 0 Undefined
180° or π 0 -1 0
-1 0 Undefined

Method for memorizing common values ​​

One method то, что может помочь в запоминании общих тригонометрических значений, состоит в том, чтобы выразить все значения sin(θ) в виде дробей, содержащих квадратный корень. Начиная с 0° и продвигаясь через 90°, sin(0°) = 0 = . Последующие значения, sin(30°), sin(45°), sin(60°) и sin(90°) следуют шаблону, так что, используя значение sin(0°) в качестве эталона, чтобы найти значения синуса для последующих углов, мы просто увеличиваем число под радикальным знаком в числителе на 1, как показано ниже.

θ 30° 45° 60° 90°
sin(θ)

Значения синуса от 0° до -90° следует той же схеме, за исключением того, что значения отрицательные, а не положительные, поскольку синус отрицателен в квадранте IV. Этот шаблон периодически повторяется для соответствующих угловых измерений, и мы можем определить значения sin(θ) на основе положения θ в единичной окружности, принимая во внимание знак синуса: синус положительный в квадрантах I и II и отрицательный в квадрантах III и IV.

Аналогичный метод запоминания можно использовать для косинуса. Начиная с 0° и заканчивая 90°, cos⁡(0°)=1=. Последующие значения cos(30°), cos(45°), cos(60°) и cos(90°) следуют шаблону, так что, используя значение cos(0°) в качестве эталона, найти значения косинуса для последующих углов, мы просто уменьшаем число под знаком радикала в числителе на 1, как показано ниже:

θ 30° 45° 60° 90°
соз(θ)

От 90° до 180° вместо этого мы увеличиваем число под радикалом на 1, но также должны учитывать квадрант, в котором находится угол. Косинус отрицателен в квадрантах II и III, поэтому значения будут равны но отрицательный. В квадрантах I и IV значения будут положительными. Этот шаблон периодически повторяется для соответствующих угловых измерений.

Пока мы помним эти значения, можно использовать соотношение

См. также синус, косинус, тангенс.


Тригонометрия — Единичный круг

Поскольку тригонометрические отношения не зависят от размера треугольника, всегда можно использовать прямоугольный треугольник, в котором длина гипотенузы равна единице. Такой треугольник можно расположить в декартовой системе таким образом, что одна его вершина будет лежать на окружности с радиусом единица. Окружность, имеющая радиус один называется единичным кругом . Когда гипотенуза равна единице, значения синуса и косинуса равны:

\[\sin\alpha = \frac{\text{opp}}{\text{hyp}}=\text{opp} \hspace{1cm} \cos \alpha = \frac{\text{adj}}{ \текст{гип}}=\текст{прил.}\]

Загрузить рабочий лист GeoGebra

Перетащите красную точку!

Как видно из рабочего листа, также верно, что \(\sin \alpha = y(A)\) и \(\cos \alpha = x(A)\), где \(x(A)\) и \(y(A)\) — это \(x\)- и \(y\)-координата соответственно \(A\). Используя координаты \(A\), вместо отношения сторон в прямоугольном треугольнике мы можем расширить определение синуса и косинуса, чтобы они были определены для всех углов. 9\круг\).

Если \(f(x)\) периодическая функция с периодом \(P\), то \(f(x)=f(x+P)\) для всех \(x\).

Построение тригонометрических функций с использованием градусов в GeoGebra

Вы можете заметить, что шкала по оси \(x\) не соответствует к углу. Причина этого в том, что GeoGebra использует радианы вместо градусов для углов. Если вы хотите, чтобы GeoGebra использовала градусы, вы должны ввести символ градуса, нажать Ctrl + O для записи символа градуса. Введенный код должен быть sin(x°) или cos(x°) .

Изменение масштаба оси \(x\) нажав Shift , а затем перетащите ось \(x\). В окне свойств для графического представления вы можете изменить расстояние между метками и добавить символ градуса как единицу измерения.

Расширение определения тангенса и котангенса

Используя определение треугольника, тангенс и котангенс определяются как

\[\tan \alpha = \frac{\text{opp}}{\text{adj}} \hspace{1cm} \cot \alpha = \frac{\text{adj}}{\text{opp}}\ ]

Загрузить рабочий лист GeoGebra

Перетащите красную точку!

Используя подобные треугольники, вы можете показать, что конструкция на листе верна для углов, образующих прямоугольный треугольник. \circ \). 9\circ, n \in \mathbb{Z} \).

радиан

Загрузить рабочий лист GeoGebra

Координаты x задаются углом. Угловая единица — радиан.

На приведенном выше листе \(x\)-координаты синей, зеленой и оранжевой точек задаются углом. В качестве видно по меткам на оси \(x\), один оборот соответствует не 360, а \(2\pi\). Другими словами, один оборот соответствует полному пути, пройденному красной точкой, т. е. длина окружности единичного круга.

При использовании радиан в качестве единицы измерения углов,
один оборот соответствует \(2\pi\).

Вы можете изменить единицу измерения угла на радианы в GeoGebra в разделе Options->Settings... , затем выберите вкладку Advanced .

Упражнения

Упражнение 1

Дуги

  • Проведите единичную окружность через две точки \(A=(0,0)\) и \(B=(1,0)\).
  • Введите точку \(C\) на окружности и отметьте угол \(\угол BAC\).
  • Убедитесь, что радианы выбрано на вкладке Advanced в разделе Options->Settings... .
  • Используйте инструмент Дуга окружности в точках \(A, B, C\).
  • Изменить радиус окружности и угол. Обратите внимание на длину дуги.

Какова длина дуги с точки зрения радиуса \(r\) и угла \(\alpha\), если радианы используются в качестве угловая единица?

Чему равна длина, если в качестве единицы угла используются градусы?

Упражнение 2

сектора

  • Удалить дугу.
  • Используйте инструмент Круговой сектор в точках \(A, B, C\).
  • Изменить радиус окружности и угол. Обратите внимание на площадь сектора.

Чему равна площадь сектора по радиусу \(r\) и углу \(\alpha\), если в качестве угловая единица?

Чему равна площадь, если в качестве единицы угла используются градусы?

Малин Кристерссон в рамках Creative Commons Attribution-Noncommercial-Share Аналогично 2. 5 Швеция Лицензия

www.malinc.se

 

APC Функции синуса и косинуса

Мотивирующие вопросы

  • Что такое функции синуса и косинуса и как они возникают из точки, пересекающей единичный круг?

  • Какими важными свойствами обладают функции синуса и косинуса?

  • Как точно или приблизительно вычислить значения \(\sin(t)\) и \(\cos(t)\text{,}\)?

В разделе 2.1 мы увидели, как отслеживание высоты точки, пересекающей окружность, генерирует периодическую функцию, как показано на рисунке 2.1.10. Затем в разделе 2.2 мы определили набор \(16\) особых точек на единичной окружности, как показано на рисунке 2.3.1.

Рисунок 2.3.1. Единичный круг с помеченными \(16\) особыми точками.

Вы также можете использовать файл Desmos по адресу http://gvsu.edu/s/0xt  1  для обзора и изучения особых точек на единичной окружности.

Предварительный просмотр 2.3.1.

Если мы рассмотрим единичный круг на рисунке 2. 3.1, начнем с \(t = 0\text{,}\) и пройдем круг против часовой стрелки, мы можем увидеть высоту, \(h\text{,}\) точка пересечения как функция угла, \(t\text{,}\) в радианах. Оттуда мы можем построить получившиеся \((t,h)\) упорядоченные пары и соединить их, чтобы создать круговую функцию, изображенную на рисунке 2.3.2.

Рисунок 2.3.2. График круговой функции, которая отслеживает высоту точки, пересекающей единичный круг.
  1. Каково точное значение \(f( \frac{\pi}{4} )\text{?}\) для \(f( \frac{\pi}{3} )\text{?} \)

  2. Заполните следующую таблицу точными значениями \(h\), которые соответствуют указанным входам.

    Таблица 2.3.3.

    \(т\) \(0\) \(\ гидроразрыва {\pi}{6}\) \(\ гидроразрыва {\pi}{4}\) \(\frac{\pi}{3}\) \(\ гидроразрыва {\pi}{2}\) \(\frac{2\pi}{3}\) \(\frac{3\pi}{4}\) \(\frac{5\pi}{6}\) \(\пи\)
    \(ч\)
    \(т\) \(\пи\) \(\frac{7\pi}{6}\) \(\frac{5\pi}{4}\) \(\frac{4\pi}{3}\) \(\frac{3\pi}{2}\) \(\frac{5\pi}{3}\) \(\frac{7\pi}{4}\) \(\frac{11\pi}{6}\) \(2\пи\)
    \(ч\)

  3. Каково точное значение \(f( \frac{11\pi}{4} )\text{?}\) для \(f( \frac{14\pi}{3} )\text{? }\)

  4. Укажите четыре различных значения \(t\), для которых \(f(t) = -\frac{\sqrt{3}}{2}\text{. }\)

Подраздел 2.3.1 Определение функции синуса

Круговая функция, которая отслеживает высоту точки на единичной окружности, проходящей против часовой стрелки от \((1,0)\) как функцию соответствующего центрального угла (в радианах), является одной из самых важных функций в математике. Таким образом, мы даем функции имя: функция sine .

Определение 2.3.4.

Дан центральный угол в единичной окружности, который измеряет \(t\) радиан и пересекает окружность в точках \((1,0)\) и \((a,b)\text{,}\), как показано на рисунке 2.3.5 мы определяем синус \(t\) , обозначенный \(\sin(t)\text{,}\) по правилу

\begin{уравнение*} \sin(t) = b\text{.} \end{equation*}

Рисунок 2.3.5. Определение синуса угла \(t\text{.}\)

Из-за соответствия между мерой угла в радианах и расстоянием, пройденным на единичной окружности, мы можем также думать о \(\sin(t)\) как об идентификации \(y\)-координаты точки после того, как она прошла \( t\) единиц против часовой стрелки по окружности от \((1,0)\text{. }\) Обратите внимание, что мы можем рассматривать синус отрицательных входных данных: например, \(\sin(-\frac{\pi} {2}) = -1\текст{.}\)

Основываясь на нашей предыдущей работе с единичным кругом, мы знаем много различных точных значений синусоидальной функции и суммируем их в Таблице 2.3.6.

Таблица 2.3.6.

\(т\) \(0\) \(\ гидроразрыва {\pi}{6}\) \(\ гидроразрыва {\pi}{4}\) \(\ гидроразрыва {\pi}{3}\) \(\ гидроразрыва {\pi}{2}\) \(\frac{2\pi}{3}\) \(\frac{3\pi}{4}\) \(\frac{5\pi}{6}\) \(\пи\)
\(\грех(т)\) \(0\) \(\ гидроразрыва{1}{2}\) \(\ гидроразрыва {\ sqrt {2}} {2} \) \(\ гидроразрыва {\ sqrt {3}} {2} \) \(1\) \(\ гидроразрыва {\ sqrt {3}} {2} \) \(\ гидроразрыва {\ sqrt {2}} {2} \) \(\ гидроразрыва{1}{2}\) \(0\)
\(т\) \(\пи\) \(\frac{7\pi}{6}\) \(\frac{5\pi}{4}\) \(\frac{4\pi}{3}\) \(\frac{3\pi}{2}\) \(\frac{5\pi}{3}\) \(\frac{7\pi}{4}\) \(\frac{11\pi}{6}\) \(2\пи\)
\(\sin(t)\) \(0\) \(-\frac{1}{2}\) \(-\frac{\sqrt{2}}{2}\) \(-\frac{\sqrt{3}}{2}\) \(-1\) \(-\frac{\sqrt{3}}{2}\) \(-\frac{\sqrt{2}}{2}\) \(-\frac{1}{2}\) \(0\)

Более того, если теперь мы нанесем эти точки обычным способом, как мы это делали в предварительном просмотре 2. 3.1, мы получим знакомую круговую волновую функцию, которая получается при отслеживании высоты точки, пересекающей окружность. Мы часто называем график на рисунке 2.3.7 синусоида .

Рисунок 2.3.7. График функции синуса на интервале \([-\frac{\pi}{4}, \frac{7\pi}{4}]\text{.}\)

Подраздел 2.3.2 Определение функции косинуса

Для любого центрального угла в радианах \(t\) в единичной окружности с одной стороной, проходящей через точку \((1,0)\text{,}\), угол порождает единственную точку \((a,b )\), лежащий на окружности. Точно так же, как мы можем рассматривать \(y\)-координату как функцию \(t\text{,}\), \(x\)-координата также является функцией \(t\text{.}\) Поэтому мы даем следующее определение.

Определение 2.3.8.

Дан центральный угол в единичной окружности, который измеряет \(t\) радиан и пересекает окружность в точках \((1,0)\) и \((a,b)\text{,}\), как показано на рисунке 2.3.9 мы определяем косинус для \(t\) , обозначаемый \(\cos(t)\text{,}\) по правилу

\begin{уравнение*} \cos(t) = а\текст{. } \end{equation*}

Рисунок 2.3.9. Определение косинуса угла \(t\text{.}\)

Опять же, из-за соответствия между радианной мерой угла и длиной дуги вдоль единичной окружности, мы можем рассматривать значение \(\cos(t)\) как отслеживание \(x\)-координаты точки, пересекающей единичный круг по часовой стрелке на расстоянии \(t\) единиц вдоль круга от \((1,0)\text{.}\) Теперь мы используем данные и информацию, которые мы получили о единичном круге, чтобы построить таблицу значения \(\cos(t)\), а также график кривой, которую он генерирует.

Мероприятие 2.3.2.

Пусть \(k = g(t)\) будет функцией, которая отслеживает координату \(x\) точки, пересекающей единичный круг против часовой стрелки от \((1,0)\text{.}\). То есть , \(g(t) = \cos(t)\text{.}\) Используйте информацию, известную нам о единичном круге, которая представлена ​​на рисунке 2.3.1, чтобы ответить на следующие вопросы.

  1. Каково точное значение \(\cos(\frac{\pi}{6})\text{?}\) из \(\cos(\frac{5\pi}{6})\text {?}\) \(\cos(-\frac{\pi}{3})\text{?}\)

  2. Заполните следующую таблицу точными значениями \(k\), которые соответствуют указанным входам.

    Таблица 2.3.10.

    \(т\) \(0\) \(\ гидроразрыва {\pi}{6}\) \(\ гидроразрыва {\pi}{4}\) \(\ гидроразрыва {\pi}{3}\) \(\ гидроразрыва {\pi}{2}\) \(\frac{2\pi}{3}\) \(\frac{3\pi}{4}\) \(\frac{5\pi}{6}\) \(\пи\)
    \(к\) г.
    \(т\) \(\пи\) \(\frac{7\pi}{6}\) \(\frac{5\pi}{4}\) \(\frac{4\pi}{3}\) \(\frac{3\pi}{2}\) \(\frac{5\pi}{3}\) \(\frac{7\pi}{4}\) \(\frac{11\pi}{6}\) \(2\пи\)
    \(к\)

  3. На осях, показанных на рисунке 2.3.11, нарисуйте точный график \(k = \cos(t)\text{.}\) Отметьте точное расположение нескольких ключевых точек на кривой.

    Рисунок 2.3.11. Оси построения \(k = \cos(t)\text{.}\)
  4. Каково точное значение \(\cos( \frac{11\pi}{4} )\text{?}\) из \(\cos( \frac{14\pi}{3} )\text {?}\)

  5. Приведите четыре различных значения \(t\), для которых \(\cos(t) = -\frac{\sqrt{3}}{2}\text{.}\)

  6. Чем график \(k = \cos(t)\) отличается от графика \(h = \sin(t)\text{?}\) Чем графики похожи?

Когда мы работаем с функциями синуса и косинуса, всегда полезно помнить их определения в терминах единичной окружности и движения точки, пересекающей окружность. На http://gvsu.edu/s/0xe  2  вы можете изучить и изучить полезную анимацию Desmos , которая показывает, как это движение по кругу генерирует каждый из соответствующих графиков.

Подраздел 2.3.3 Свойства функций синуса и косинуса

Поскольку функция синуса является результатом отслеживания координаты \(y\) точки, пересекающей единичный круг, а функция косинуса — координаты \(x\), эти две функции имеют несколько общих свойств круговых функций.

Свойства функций синуса и косинуса.

Для обоих \(f(t) = \sin(t)\) и \(g(t) = \cos(t)\text{,}\)

  • домен функции — все действительные числа;

  • диапазон функции: \([-1,1]\text{;}\)

  • средняя линия функции равна \(y = 0\text{;}\)

  • амплитуда функции равна \(a = 1\text{;}\)

  • период функции равен \(p = 2\pi\text{.}\)

Также полезно сопоставить графики функций синуса и косинуса на одних и тех же координатных осях. Когда мы это делаем, как показано на рисунке 2.3.12, мы видим, что кривые можно рассматривать как горизонтальные переносы друг друга.

Рисунок 2.3.12. Графики функций синуса и косинуса.

В частности, поскольку график синуса можно рассматривать как график косинуса, сдвинутый на \(\frac{\pi}{2}\) единиц вправо, отсюда следует, что для любого значения \(t\text{,}\ )

\begin{уравнение*} \sin(t) = \cos(t-\frac{\pi}{2})\text{.} \end{уравнение*}

Аналогично, поскольку график косинуса можно рассматривать как график синуса, сдвинутый влево,

\begin{уравнение*} \cos(t) = \sin(t + \frac{\pi}{2})\text{. } \end{уравнение*} 92(т) = 1\текст{.} \end{equation*}

На графиках двух функций есть дополнительные тенденции и закономерности, которые мы рассмотрим подробнее в следующем упражнении.

Мероприятие 2.3.3.

Используйте рисунок 2.3.12, чтобы помочь ответить на следующие вопросы.

  1. Приведите пример наибольшего из найденных интервалов, на котором \(f(t) = \sin(t)\) убывает.

  2. Приведите пример наибольшего интервала, который вы можете найти, на котором \(f(t) = \sin(t)\) убывает и вогнут вниз.

  3. Приведите пример наибольшего из найденных интервалов, на котором \(g(t) = \cos(t)\) возрастает.

  4. Приведите пример наибольшего интервала, который вы можете найти, на котором \(g(t) = \cos(t)\) возрастает и вогнут вверх.

  5. Без каких-либо вычислений, на каком интервале средняя скорость изменения \(g(t) = \cos(t)\) больше: \([\pi, \pi+0,1]\) или \([ \frac{3\pi}{2}, \frac{3\pi}{2} + 0. 1]\text{?}\) Почему?

  6. Как бы вы в целом охарактеризовали места на графиках синуса и косинуса, где функции возрастают или убывают быстрее всего?

  7. Думая с точки зрения единичной окружности, для которой квадранты плоскости \(x\)-\(y\) \(\cos(t)\) отрицательны для угла \(t\), лежащего в этот квадрант?

Подраздел 2.3.4 Использование вычислительной техники

Мы установили, что знаем точное значение \(\sin(t)\) и \(\cos(t)\) для любого из \(t\)-значений в таблице 2.3.6, а также для любого такого \(t \pm 2j\pi\text{,}\), где \(j\) — целое число, из-за периодичности функций. Но что, если мы хотим узнать \(\sin(1.35)\) или \(\cos(\frac{\pi}{5})\) или значения для других входных данных, которых нет в таблице?

Любое стандартное вычислительное устройство, такое как научный калькулятор, Desmos , Geogebra или WolframAlpha , имеет возможность вычислять функции синуса и косинуса при любом входном сигнале, который мы пожелаем. Поскольку ввод рассматривается как угол, каждое вычислительное устройство может учитывать угол в радианах или градусах. Всегда важно знать, какой тип ввода ожидает ваше устройство. Выбранное нами вычислительное устройство — Десмос . В Desmos вы можете изменить тип ввода между радианами и градусами, щелкнув значок гаечного ключа в правом верхнем углу и выбрав нужные единицы измерения. Радианная мера используется по умолчанию.

Требуются основательные и сложные математические вычисления, чтобы вычислительное устройство могло вычислить функции синуса и косинуса при любом значении, которое мы захотим; алгоритмы включают идею исчисления, известную как бесконечный ряд. Несмотря на то, что ваше вычислительное устройство является мощным, полезно и важно понимать значение этих значений на единичной окружности и помнить специальные точки, для которых мы точно знаем выходные значения функций синуса и косинуса.

Мероприятие 2.3.4.

По возможности точно ответьте на следующие вопросы. Если вы оцениваете значение, делайте это с точностью не менее \(5\) знаков после запятой.

  1. Координата \(x\) точки на единичной окружности, лежащей в третьем квадранте и координата \(y\) которой равна \(y = -\frac{3}{4}\text{. }\)

  2. \(y\)-координата точки на единичной окружности, образованной центральным углом в стандартном положении, который измеряет \(t = 2\) радиан.

  3. \(x\)-координата точки на единичной окружности, образованной центральным углом в стандартном положении, который измеряет \(t = -3,05\) радиан.

  4. Значение \(\cos(t)\), где \(t\) — угол в квадранте II, удовлетворяющий условию \(\sin(t) = \frac{1}{2}\text{.}\ )

  5. Значение \(\sin(t)\), где \(t\) — угол в квадранте III, для которого \(\cos(t) = -0,7\text{.}\)

  6. Средняя скорость изменения \(f(t) = \sin(t)\) на интервалах \([0,1,0,2]\) и \([0,8,0,9]\текст{.}\)

  7. Средняя скорость изменения \(g(t) = \cos(t)\) на интервалах \([0. 1,0.2]\) и \([0.8,0.9]\text{.}\)

Подраздел 2.3.5 Резюме

  • Функции синуса и косинуса являются результатом отслеживания \(y\)- и \(x\)-координат точки, пересекающей единичный круг против часовой стрелки от \((1,0)\text{.}\) значением \(\sin(t)\) является \(y\)-координата точки, которая прошла \(t\) единиц по окружности от \((1,0)\) (или, что то же самое, соответствует к углу \(t\) радиан), а значение \(\cos(t)\) является \(x\)-координатой той же точки.

  • Функции синуса и косинуса являются периодическими функциями, которые имеют одну и ту же область определения (множество всех действительных чисел), диапазон (интервал \([-1,1]\)), среднюю линию (\(y = 0\) ), амплитуда (\(a = 1\)) и период (\(P = 2\pi\)). Кроме того, функция синуса представляет собой горизонтальный сдвиг функции косинуса на \(\frac{\pi}{2}\) единиц вправо, поэтому \(\sin(t) = \cos(t-\frac{\ pi}{2})\) для любого значения \(t\text{.}\)

  • Если \(t\) соответствует одному из известных нам специальных углов единичной окружности (как на рис.  2.3.1), мы можем вычислить значения \(\sin(t)\) и \(\ cos(t)\) точно. Для других значений \(t\text{,}\) мы можем использовать вычислительное устройство для оценки значения любой функции при данном входе; когда мы это делаем, мы должны позаботиться о том, в каких единицах мы вычисляем: в радианах или в градусах. 9\circ)\)

    • Отрицательный

    • Положительный

    \(\sin(\frac{15\pi}{16})\)

    • Положительный

    • Отрицательный

    \(\cos(\frac{20\pi}{21})\)

    • Отрицательный

    • Положительный

    4.

    Вопрос 30:

    a) Напишите выражение (включая переменные \(a\) и \(h\)) для наклона отрезка, соединяющего \(S\) и \(T\) на рисунке выше .

    Slope = (выражение, включающее переменные a и h )

    b) Оцените выражение для \(a = 1,6\) и \(h = 0,01\text{.}\) Округлите ответ до два десятичных знака.

    Наклон = (значение, округленное до двух знаков после запятой)

    5.

    Добавить комментарий

    Ваш адрес email не будет опубликован. Обязательные поля помечены *

    © 2015 - 2019 Муниципальное казённое общеобразовательное учреждение «Таловская средняя школа»

    Карта сайта